Sie sind auf Seite 1von 78

CBSE CLASS 12

MATHEMATICS

FOR 12TH CLASS

COMPLETE COVERAGE FOR CBSE BOARDS EXAMS 


CONCISE THEORY
PREVIOUS YEARS CBSE FULLY SOLVED
SUBJECTIVE EXAMPLES AND EXERCISES
IMPORTANT FULLY SOLVED

U.K. Gupta
M.Sc. Mathematics (Gold Medal)
Ex-Faculty (Bansal Classes Kota)
CBSE CLASS 12

MATHEMATICS

CONTENTS MODULE - 1

1. MATRICES AND DETERMINANTS


2. RELATIONS
3. FUNCTIONS
Chapter - 1
MATRICES & DETERMINANT
Introduction :
Any rectangular arrangement of numbers (real or complex) (or of real valued or complex valued
expressions) is called a matrix. If a matrix has m rows and n columns then the order of matrix is
written as m × n and we call it as order m by n
The general m × n matrix is

 a11 a12 a13 ...... a1j a1n 


.....
 
 a 21 a 22 a 23 ...... a 2 j ..... a 2n 
 ..... ..... ..... ..... ..... ..... ..... 
A=  
 ai1 ai2 a i3 ...... a ij ...... a in 
 ..... ..... ..... ..... ..... ..... ..... 
 
a m1 am 2 a m3 ..... a mj ..... a mn 

where aij denote the element of ith row & jth column. The above matrix is usually denoted as [aij]m × n .

Notes :
(i) The elements a11, a22, a33,........ are called as diagonal elements. Their sum is called as
trace of A denoted as tr(A)

(ii) Capital letters of English alphabets are used to denote matrices.

(iii) Order of a matrix : If a matrix has m rows and n columns, then we say that its order is "m by n",

STUDYPIVOT.COM
written as "m × n".

Example # 1 : Construct a 3 × 2 matrix whose elements are given by aij =


1
2
| i –3j |.

a11 a12 
 
Solution : In general a 3 × 2 matrix is given by A = a21 a22  .
a31 a32 
1
aij = | i – 3j |, i = 1, 2, 3 and j = 1, 2
2
1 1 5
Therefore a11 = | 1 – 3 × 1 | = 1 a12 = | 1 – 3 × 2 | =
2 2 2
1 1 1
a21 = |2–3×1|= a22 = |2–3×2|=2
2 2 2
1 1 3
a31 = |3–3×1|=0 a32 = |3–3×2|=
2 2 2
Hence the required matrix is given hy A =
Types of Matrices :
Row matrix :
A matrix having only one row is called as row matrix (or row vector).General form of row matrix
is A = [a11, a12, a13, ...., a1n]

This is a matrix of order "1 × n" (or a row matrix of order n)

Column matrix :
A matrix having only one column is called as column matrix (or column vector).

1
 a11 
 
 a 21 
Column matrix is in the form A = 
... 
 
a m1 
This is a matrix of order "m × 1" (or a column matrix of order m)

Zero matrix :
A = [aij]m × n is called a zero matrix, if aij = 0  i & j.

0 0 0
0 0 0  
e.g. : (i)   (ii)  0 0 0 
0 0 0
 0 0 0 

Square matrix :
A matrix in which number of rows & columns are equal is called a square matrix. The general
form of a square matrix is

 a11 a12 ....... a1n 


 
a 21 a 22 ........ a 2n 
A =  which we denote as A = [aij]n.
....... ....... ....... .......
 
 an1 a n2 ....... a nn 
This is a matrix of order "n × n" (or a square matrix of order n)

Diagonal matrix :

STUDYPIVOT.COM
A square matrix [aij]n is said to be a diagonal matrix if aij = 0 for i  j. (i.e., all the elements of
the square matrix other than diagonal elements are zero)
Note : Diagonal matrix of order n is denoted as Diag (a11, a22, ......ann).

 a 0 0 0
a 0 0  
   0 b 0 0
0 b 0
e.g. : (i)  (ii)  0 0 0 0
 0 0 c   
 0 0 0 c 

Scalar matrix :
Scalar matrix is a diagonal matrix in which all the diagonal elements are same. A = [a ij]n is a
scalar matrix, if (i) aij = 0 for i  j and (ii) aij = k for i = j.

a 0 0 
a 0  
0 a 0
e.g. : (i)   (ii)  
0 a  0 0 a 

Unit matrix (identity matrix) :


Unit matrix is a diagonal matrix in which all the diagonal elements are unity. Unit matrix of
order 'n' is denoted by n (or ).
i.e. A = [aij]n is a unit matrix when aij = 0 for i  j & aii = 1

1 0 0
 1 0  
eg. 2 =   , 3 = 0 1 0  .
0 1 0 0 1

Upper triangular matrix :


A = [aij]m × n is said to be upper triangular, if aij = 0 for i > j (i.e., all the elements below the

2
diagonal elements are zero).

a b c d a b c 
 
e.g. : (i)  0 x y z  (ii)  0 x y 
 0 0 u v   0 0 z 

Lower triangular matrix :


A = [aij]m × n is said to be a lower triangular matrix, if aij = 0 for i < j. (i.e., all the elements above
the diagonal elements are zero.)

a 0 0 a 0 0 0
   
b c 0 b c 0 0
e.g. : (i)  (ii) 
 x y z   x y z 0 

Comparable matrices : Two matrices A & B are said to be comparable, if they have the same order
(i.e., number of rows of A & B are same and also the number of columns).
2 3 4 3 4 2
e.g. : (i) A =   & B=   are comparable
 3 1 2  0 1 3

3 0
2 3 4  
4 1
e.g. : (ii) C =   & D=  are not comparable
 3 1 2   2 3 

Equality of matrices :
Two matrices A and B are said to be equal if they are comparable and all the corresponding
elements are equal.

STUDYPIVOT.COM
Let A = [aij] m × n & B = [bij]p × q
A = B iff (i) m = p, n = q
(ii) aij = bij  i & j.

 sin  1/ 2  1/ 2 sin  


   
Example # 2 : Let A =  1/ 2 cos   & B = cos  cos  . Find  so that A = B.
 cos  tan   cos  1 
   
Solution : By definition A & B are equal if they have the same order and all the corresponding elements
are equal.
1 1
Thus we have sin  = , cos = – & tan  = – 1
2 2

  = (2n + 1) – .
4

 x  3 z  4 2y – 7  0 6 3 y – 2
   
– 6 a –1 0  –6 – 3 2c  2 
Example # 3 : If  =  , then find the values of a, b, c, x, y and z.
b – 3 – 21 0  2b  4 – 21 0 
Solution : As the given matrices are equal, therefore, their corresponding elements must be equal.
Comparing the corresponding elements, we get
x+3=0 z+4=6 2y – 7 = 3y – 2
a–1=–3 0 = 2c + 2 b – 3 = 2b + 4
 a = – 2, b = – 7, c = – 1, x = – 3, y = – 5, z = 2

Multiplication of matrix by scalar :


Let  be a scalar (real or complex number) & A = [aij]m × n be a matrix. Thus the product A is

3
defined as A = [bij]m × n where bij = aij  i & j.

 2 1 3 5    6 3  9  15 
   
e.g. : A =  0 2 1 3  & – 3A  (–3) A =  0  6  3 9 
 0 0  1  2   0 0 3 6 

Note : If A is a scalar matrix, then A = , where  is a diagonal entry of A

Addition of matrices :
Let A and B be two matrices of same order (i.e. comparable matrices). Then A + B is defined to
be.
A + B = [aij]m × n + [bij]m × n.
= [cij]m × n where c ij = aij + bij  i & j.

 1 1   1 2  0 1
     
2 3  2 3 , A + B =  0 0 
e.g. : A =  , B= 
 1 0   5 7   6 7 

Substraction of matrices :
Let A & B be two matrices of same order. Then A – B is defined as A + (– B) where – B is (– 1)
B.

Properties of addition & scalar multiplication :


Consider all matrices of order m × n, whose elements are from a set F (F denote Q, R or C).
Let Mm × n (F) denote the set of all such matrices.
Then
(a) A  Mm × n (F) & B  Mm × n (F)  A + B  Mm × n(F)
(b)
(c)
(d)
STUDYPIVOT.COM
A+B= B+A
(A + B) + C = A + (B + C)
O = [o]m × n is the additive identity.
(e) For every A  Mm × n(F), – A is the additive inverse.
(f)  (A + B) = A + B
(g) A = A
(h) ( 1 +  2) A =  1A +  2A

8 0  2 – 2
   
4 – 2 4 2
Example # 4 : IF A =  and B =  , then find the matrix X, such that 2A + 3X = 5B
3 6   – 5 1
Solution : We have 2A + 3X = 5B.
 3X = 5B – 2A
1
 X= (5B – 2A)
3

  2 – 2 8 0     10 – 10   – 16 0 
1     1   
 X =  5 4 2 – 24 – 2  =   20 10    – 8 4 
3 3
 1 3 6    
5   – 6 – 12 
 – 5    – 25

 – 10 
 –2 3 
 10 – 16 – 10  0  – 6 – 10  
1  1  14 
20 – 8 10  4 12 14  4 
 X=   = 3  =  3 
3  – 13 – 7 
 – 25 – 6 5 – 12   – 31 –7 
 3 3 

Multiplication of matrices :
4
Let A and B be two matrices such that the number of columns of A is same as number of rows
of B. i.e., A = [aij]m × p & B = [bij]p × n.
p
Then AB = [c ij]m × n where cij = a
k 1
ik b kj , which is the dot product of ith row vector of A and j th

column vector of B.

0 1 1 1
1 2 3   3 4 9 1
0 0 1 0
e.g. : A =   , B=  , AB =  
2 3 1  1 1 2 0  1 3 7 2

Notes : (1) The product AB is defined iff the number of columns of A is equal to the number of rows
of B. A is called as premultiplier & B is called as post multiplier. AB is defined 
/ BA is
defined.
(2) In general AB  BA, even when both the products are defined.
(3) A (BC) = (AB) C, whenever it is defined.

Properties of matrix multiplication :


Consider all square matrices of order 'n'. Let Mn (F) denote the set of all square matrices of
order n. (where F is Q, R or C). Then
(a) A, B  Mn (F)  AB  Mn (F)
(b) In general AB  BA
(c) (AB) C = A(BC)
(d) n, the identity matrix of order n, is the multiplicative identity.
An = A = n A  A  Mn (F)
(e) For every non singular matrix A (i.e., |A|  0) of Mn (F) there exist a unique (particular)
matrix B  Mn (F) so that AB = n = BA. In this case we say that A & B are multiplicative
inverse of one another. In notations, we write B = A–1 or A = B–1.
(f)
(g)
(h)
STUDYPIVOT.COM
If  is a scalar (A) B = (AB) = A(B).
A(B + C) = AB + AC  A, B, C  Mn (F)
(A + B) C = AC + BC  A, B, C  Mn (F).

Notes : (1) Let A = [aij]m × n. Then An = A & m A = A, where n & m are identity matrices of order
n & m respectively.
(2) For a square matrix A, A2 denotes AA, A3 denotes AAA etc.

 1 2 3
 
3 – 2 1
Example # 5 : If A =  , then show that A3 – 23A – 40 I = O
4 2 1

 1 2 3  1 2 3 19 4 8 
     
3 – 2 1 3 – 2 1 =  1 12 8 
Solution : We have A2 = A.A =   
4 2 1 4 2 1 14 6 15 

 1 2 3 19 4 8  63 46 69


     
3 – 2 1  1 12 8  69 – 6 23
So A3 = AA2 =  = 
4 2 1 14 6 15  92 46 63

63 46 69  1 2 3 1 0 0
     
69 – 6 23 3 – 2 1 0 1 0
Now A3 – 23A – 40I =  – 23  – 40 
92 46 63 4 2 1 0 0 1

63 46 69  – 23 – 46 – 69  – 40 0 0 


     
69 – 6 23 – 69 46 – 23 0 – 40 0 
=  +  + 
92 46 63  – 92 – 46 – 23  0 0 – 40

5
63 – 23 – 40 46 – 46  0 69 – 69  0 
 
69 – 69  0 – 6  46 – 40 23 – 23  0 
= 
 90 – 92  0 46 – 46  0 63 – 23 – 40

0 0 0
 
0 0 0
=  =O
0 0 0

Self practice problems :

cos   sin 
(1) If A() =   , verify that A() A() = A( + ).
 sin  cos  
Hence show that in this case A(). A() = A() . A().

4 6  1  2 4
   
(2) Let A =  3 0 2  , B =  0 1 and C = [3 1 2].
 1  2 5   1 2 
Then which of the products ABC, ACB, BAC, BCA, CAB, CBA are defined. Calculate the product
whichever is defined.
Answer (2) Only CAB is defined. CAB = [25 100]

Transpose of a matrix :
Let A =[aij]m × n. Then the transpose of A is denoted by A( or AT) and is defined as

STUDYPIVOT.COM
A = [bij]n × m where bij = aji  i & j.
i.e. A is obtained by rewriting all the rows of A as columns (or by rewriting all the columns of A as
rows).

1 a x 
1 2 3 4   

a b c d
 2 b y 
e.g. : A =   , A =  3 c z 
 x y z w   
4 d w 

Results : (i) For any matrix A = [aij]m × n, (A) = A


(ii) Let  be a scalar & A be a matrix. Then (A) = A
(iii) (A + B) = A + B & (A – B) = A – B for two comparable matrices A and B.
(iv) (A1 ± A2 ± ..... ± An) = A1 ± A2 ± ..... ± An, where Ai are comparable.
(v) Let A = [aij]m × p & B = [bij]p × n , then (AB) = BA
(vi) (A1 A2 .......An)= An. An – 1 ...........A2 . A1, provided the product is defined.

Symmetric & skew-symmetric matrix : A square matrix A is said to be symmetric if A = A


i.e. Let A = [aij]n. A is symmetric iff aij = aji  i & j.
A square matrix A is said to be skew-symmetric if A = – A
i.e. Let A = [aij]n. A is skew-symmetric iff aij = – aji  i & j.

a h g
 
e.g. A = h b f  is a symmetric matrix.
g f c 

6
o x y
 
B =  x o z  is a skew-symmetric matrix.
 y  z 0 
Notes : (1) In a skew-symmetric matrix all the diagonal elements are zero.
( aii = – aii  aii = 0)
(2) For any square matrix A, A + A is symmetric & A – A is skew-symmetric.
(3) Every square matrix can be uniqualy expressed as a sum of two square matrices of
which one is symmetric and the other is skew-symmetric.
1 1
A = B + C, where B = (A + A) & C = (A – A).
2 2

 – 2
 
4
Example # 6 : If A =   , B = [1 3 – 6], verify that (AB)' = B'A'.
 5
Solution : We have

 – 2
 
4
A =   , B = [1 3 –6]
 5 

 – 2  – 2 – 6 12 
   
4 4 12 – 24
Then AB =   [1 3 –6] = 
 5  5 15 – 30 

 1

STUDYPIVOT.COM
 
3
Now A' = [–2 4 5], B' =  
 – 6

 1 – 2 4 5
   
3 – 6 12 15
B'A' =   [–2 4 5] =   = (AB)'
 – 6   12 – 24 – 30 
Clearly (AB)' = B'A'

 2 – 2 – 4
 
–1 3 4
Example # 7 : Express the matrix B =  as the sum of a symmetric and a skew symmetric
 1 – 2 – 3
matrix.

 2 –1 1
 
– 2 3 – 2
Solution : Here B' = 
 – 4 4 – 3

 3 3
 4 – 3 – 3  2 –
2

2
 3 
1 1 – 3 6 2

Let P= (B + B') =  = – 2 3 1 
2 2 – 3 2 – 6  
  3 
– 2 1 – 3
 

7
 –3 – 3
 2 2 2 
– 3 
Now P' =  3 1  =P
 2 
– 3 
 2 1 – 3
 

1
Thus P= (B + B') is a symmetric matrix.
2

 –1 – 5
0 – 1 – 5 0 2 2 
1 
1 1 1 0 
Also, Let Q = (B – B') =  6  = 2 0 3 
2 2 5 – 6 0   
  5 
2 –3 0 
 
 1 5 
 0 2 3 
 1 
Now Q' =  – 0 – 3 = – Q
 2 
 5 
– 2 3 0 
 

1
Thus Q= (B – B') is a skew symmetric matrix.
2

 –3 – 3  –1 – 5
 2 0 2 
STUDYPIVOT.COM 2 2  2  2 – 2 – 4
– 3  1   
Now P+Q=  3 1  +  0 3  = – 1 3 4 
=B
 2  2   1 – 2 – 3 
– 3  5 
1 – 3 2 –3 0 
 2  
 
Thus, B is reresented as the sum of a symmetric and a skew symmetric matrix.
Example # 8 : Show that BAB is symmetric or skew-symmetric according as A is symmetric or skew-
symmetric (where B is any square matrix whose order is same as that of A).
Solution : Case - A is symmetric  A = A
(BAB) = (B)AB = BAB  BAB is symmetric.
Case -  A is skew-symmetric  A = – A
(BAB) = (B)AB
= B ( – A) B
= – (BAB)
 BAB is skew-symmetric
Self practice problems :
(3) For any square matrix A, show that AA & AA are symmetric matrices.
(4) If A & B are symmetric matrices of same order, then show that AB + BA is symmetric and
AB – BA is skew-symmetric.
Submatrix : Let A be a given matrix. The matrix obtained by deleting some rows or columns of A is called
as submatrix of A.
a b c d 
 
eg. A = x y z w 
p q r s 
a c  a b c 
  a b d  
Then  x z , p q s  ,  x y z are all submatrices of A.
p r    p q r 

8
Determinant of a square matrix :
To every square matrix A = [aij] of order n, we can associate a number (real or complex) called determinant
of the square matrix.
Let A = [a]1×1 be a 1×1 matrix. Determinant A is defined as |A| = a.
e.g. A = [– 3]1×1 |A| = – 3
a b
Let A =   , then |A| is defined as ad – bc.
 c d
 5 3
e.g. A=   , |A| = 23
 1 4 
Minors & Cofactors :
Let  be a determinant. Then minor of element aij, denoted by Mij, is defined as the determinant
of the submatrix obtained by deleting ith row & j th column of . Cofactor of element aij, denoted
by C ij, is defined as Cij = (– 1)i + j Mij.
a b
e.g. 1 =
c d
M11 = d = C 11
M12 = c, C12 = – c
M21 = b, C21 = – b
M22 = a = C 22
a b c
e.g. 2 = p q r
x y z

q r

STUDYPIVOT.COM
M11 = = qz – yr = C111.
y z

a b
M23 = = ay – bx, C23 = – (ay – bx) = bx – ay etc.
x y

Determinant of any order : Let A = [aij]n be a square matrix (n > 1). Determinant of A is defined as the
sum of products of elements of any one row (or any one column) with
corresponding cofactors.

a11 a12 a13 


 
e.g.1 A = a 21 a 22 a 23 
a 31 a 32 a 33 
|A| = a11C11 + a12 C12 + a13C13 (using first row).
a 22 a 23 a 21 a 23 a 21 a 22
= a11 – a12 + a13
a 32 a 33 a 31 a 33 a 31 a 32
|A| = a12 C12 + a22 C22 + a32C32 (using second column).
a 21 a 23 a11 a13 a11 a13
= – a12 + a22 – a32 .
a 31 a 33 a 31 a 33 a 21 a 23

Transpose of a determinant : The transpose of a determinant is the determinant of transpose of the


corresponding matrix.
a1 b1 c1 a1 a 2 a3
T
D = a2 b2 c2  D  b1 b 2 b3
a3 b3 c3 c1 c 2 c3

9
Properties of determinant :
(1) |A| = |A| for any square matrix A.

i.e. the value of a determinant remains unaltered, if the rows & columns are inter changed,

a1 b1 c1 a1 a 2 a3
i.e. D = a2 b2 c 2  b1 b 2 b 3 = D
a3 b3 c3 c1 c 2 c3
(2) If any two rows (or columns) of a determinant be interchanged, the value of determinant
is changed in sign only.

a1 b1 c1 a2 b2 c2
e.g. Let D1 = a 2 b2 c2 & D2 = a1 b1 c 1 Then D2 = – D1
a3 b3 c3 a 3 b3 c 3

(3) Let  be a scalar. Than  |A| is obtained by multiplying any one row (or any one column)
of |A| by 
a1 b1 c1 Ka1 Kb1 Kc1
D = a2 b2 c2 and E = a 2 b2 c2 Then E= KD
a 3 b3 c3 a3 b3 c3
(4) | AB | = | A | | B |.

(5) |A| =  n |A|, when A = [aij]n.

(6) A skew-symmetric matrix of odd order has deteminant value zero.

(7)
STUDYPIVOT.COM
If a determinant has all the elements zero in any row or column, then its value is zero,
0 0 0
i.e. D = a 2 b2 c2 = 0.
a 3 b3 c3
(8) If a determinant has any two rows (or columns) identical (or proportional), then its value
is zero,
a1 b1 c1
i.e. D = a1 b1 c 1 = 0.
a 3 b3 c 3

(9) If each element of any row (or column) can be expressed as a sum of two terms then
the determinant can be expressed as the sum of two determinants, i.e.
a1x b1y c1z a1 b1 c1 x y z
a2 b2 c2  a2 b2 c 2  a2 b2 c2
a3 b3 c3 a3 b3 c3 a3 b3 c3
(10) The value of a determinant is not altered by adding to the elements of any row (or
column) a constant multiple of the corresponding elements of any other row (or column),

a1 b1 c1 a1  ma 2 b1  mb 2 c 1  mc 2
i.e. D1 = a 2 b2 c2 and D2 = a2 b2 c2 . Then D2= D1
a3 b3 c3 a 3  na1 b3  nb1 c 3  nc 1

(11) Let A = [aij]n. The sum of the products of elements of any row with corresponding
cofactors of any other row is zero. (Similarly the sum of the products of elements of
any column with corresponding cofactors of any other column is zero).

10
a b c
Example # 9 Simplify b c a
c a b
Solution : Let R1  R1 + R2 + R3

abc abc abc 1 1 1


 b c a = (a + b + c) b c a
c a b c a b
Apply C 1 C 1 – C 2, C 2  C 2 – C 3

0 0 1
= (a + b + c) b  c c  a a
c a a b b
= (a + b + c) ((b – c) (a – b) – (c – a)2)
= (a + b + c) (ab + bc – ca – b2 – c 2 + 2ca – a2)
= (a + b + c) (ab + bc + ca – a2 – b2 – c 2)  3abc – a3 – b3 – c3
a b c
Example # 10 Simplify a2 b2 c2
bc ca ab
Solution : Given detereminant is equal to

a2 b2 c2 a2 b2 c2
1 abc
= a3 b3 c3 = a3 b3 c3
abc abc
abc abc abc 1 1 1

STUDYPIVOT.COM
Apply C1  C1 – C2,

a2  b2 b2  c 2 c2
C2  C2 – C3

3 3 3 3
= a b b c c3
0 0 1

ab bc c2
2 2
= (a – b) (b – c) a  ab  b b 2  bc  c 2 c3
0 0 1

= (a – b) (b – c) [ab2 + abc + ac2 + b3 + b2C + bc 2 – a2b – a2c – ab2 – abc – b3 – b2c]


= (a – b) (b – c) [c(ab + bc + ca) – a(ab + bc + ca)]
= (a – b) (b – c) (c – a) (ab + bc + ca)

Self practice problems


0 ba c a
(5) Find the value of  = a  b 0 c b .
ac bc 0

b 2  ab b  c bc  ac
2 2
(6) Simplify ab  a a  b b  ab .
bc  ac c  a ab  a 2

ab c 2a 2a
2b bc a 2b
(7) Prove that = (a + b + c) 3.
2c 2c c ab

11
1 a bc
(8) Show that 1 b ca = (a – b) (b – c) (c – a) by using factor theorem .
1 c ab
Answers : (5) 0 (6) 0

Application of determinants : Following examples of short hand writing large expressions are:

(i) Area of a triangle whose vertices are (x r, yr); r = 1, 2, 3 is:


x1 y1 1
1 x2 y2 1
D= If D = 0 then the three points are collinear.
2 x y 1
3 3

x y 1
(ii) Equation of a straight line passing through (x 1, y1) & (x 2, y2) is x1 y1 1 = 0
x2 y2 1
(iii) The lines: a1x + b1y + c1 = 0........ (1)
a2x + b2y + c2 = 0........ (2)
a3x + b3y + c3 = 0........ (3)
a1 b1 c1
are concurrent if, a2 b2 c 2 = 0.
a3 b3 c3
Condition for the consistency of three simultaneous linear equations in 2 variables.
(iv) ax² + 2 hxy + by² + 2 gx + 2 fy + c = 0 represents a pair of straight lines if:
a h g

STUDYPIVOT.COM
abc + 2 fgh  af²  bg²  ch² = 0 = h b f
g f c

Singular & non singular matrix : A square matrix A is said to be singular or non-singular according
as |A| is zero or non-zero respectively.
Cofactor matrix & adjoint matrix : Let A = [aij] n be a square matrix. The matrix obtained by
replacing each element of A by corresponding cofactor is called as
cofactor matrix of A, denoted as cofactor A. The transpose of cofactor
matrix of A is called as adjoint of A, denoted as adj A.
i.e. if A = [aij]n
then cofactor A = [c ij]n when c ij is the cofactor of aij  i & j.
Adj A = [dij]n where dij = c ji  i & j.
Properties of cofactor A and adj A:
(a) A . adj A = |A| n = (adj A) A where A = [aij]n.
(b) |adj A| = |A|n – 1, where n is order of A.
In particular, for 3 × 3 matrix, |adj A| = |A|2
(c) If A is a symmetric matrix, then adj A are also symmetric
matrices.
(d) If A is singular, then adj A is also singular.
Example # 11 : For a 3×3 skew-symmetric matrix A, show that adj A is a symmetric matrix.

0 a b  c2  bc ca 
   
A =  a 0 c 
2
Solution : cof A =  bc b  ab 
  b  c 0   ca  ab a 2 
 

 c2  bc ca 
 2 
adj A = (cof A) =  bc b  ab  which is symmetric.
 ca  ab a 2 
 
12
Inverse of a matrix (reciprocal matrix) :
1
Let A be a non-singular matrix. Then the matrix adj A is the
|A|
multiplicative inverse of A (we call it inverse of A) and is denoted by A–1.
We have A (adj A) = |A| n = (adj A) A
 1   1 
 A  adj A  = n =  adj A  A, for A is non-singular
 | A |   | A | 
1
 A–1 = adj A.
|A|
Remarks :
1. The necessary and sufficient condition for existence of inverse of A is that A is non-singular.
2. A–1 is always non-singular.
3. If A = dia (a11, a22, ....., ann) where aii  0  i, then A–1 = diag (a11– 1, a22–1, ...., ann–1).
4. (A–1) = (A)–1 for any non-singular matrix A. Also adj (A) = (adj A).
5. (A–1)–1 = A if A is non-singular.
1 –1
6. Let k be a non-zero scalar & A be a non-singular matrix. Then (kA)–1 = A .
k
1
7. |A–1| = | A | for |A|  0.
8. Let A be a non-singular matrix. Then AB = AC  B = C & BA = CA  B= C.
9. A is non-singular and symmetric  A–1 is symmetric.
10. (AB)–1 = B–1 A–1 if A and B are non- singular.

STUDYPIVOT.COM
11. In general AB = 0 does not imply A = 0 or B = 0. But if A is non-singular and AB = 0, then B = 0.
Similarly B is non-singular and AB = 0  A = 0. Therefore, AB = 0  either both are singular or one of
them is 0.

1 3 3
 
1 4 3
Example # 12 : If A =  , then verify that A adj A = | A | . Also find A–1
1 3 4
Solution : We have | A | = 1 (16 – 9) – 3 (4 – 3) + 3 (3 – 4) = 1  0
Now A11 = 7, A12 = – 1, A13 = – 1, A21 = – 3, A22 = 1, A23 = 0,A31 = – 3, A32 = 0, A33 = 1

 7 – 3 – 3
 
–1 1 0 
Therefore adj A = 
 – 1 0 1 

1 3 3   7 – 3 – 3 7 – 3 – 3 – 3  3  0 – 3  0  3 
    
1 4 3 – 1 1 0  7 – 4 – 3 – 3  4  0 – 3  0  3
Now A(adj A) =  = 
1 3 4  – 1 0 1  7 – 3 – 4 – 3  3  0 – 3  0  4

1 0 0 1 0 0
   
0 1 0 0 1 0
=  = (1)  = |A|. I
0 0 1 0 0 1

 7 – 3 – 3  7 – 3 – 3
1 1    
Also A–1 adj A = – 1 1 0 
= 
–1 1 0 
|A| 1  – 1 0 1   – 1 0 1 

13
2 3 
Example # 13 : Show that the matrix A =   satisfies the equation A2 – 4A + I = O, where I is 2 × 2 identity
 1 2
matrix and O is 2 × 2 zero matrix. Using the equation, find A–1 .
2 3  2 3  7 12
Solution : We have A2 = A.A =     =  
 1 2  1 2 4 7 

7 12 8 12  1 0 0 0
Hence A2 – 4A + I =   –   +   =   =0
4 7  4 8  0 1 0 0
Now A2 – 4A + I = 0
Therefore A A – 4A = – I
or AA(A–1) – 4 A A–1 = – I A–1 (Post multiplying by A–1 because |A| 0)
or A (A A–1) – 4I = – A–1
or AI – 4I = – A–1
4 0  2 3   2 – 3
or A–1 = 4I – A =   –   =  
0 4   1 2 – 1 2 

 2 – 3
Hence A–1 =  
– 1 2 

Example # 14 : For two non-singular matrices A & B, show that adj (AB) = (adj B) (adj A)
Solution : We have (AB) (adj (AB)) = |AB| n
= |A| |B| n
A (AB)(adj (AB)) = |A| |B| A–1
–1

STUDYPIVOT.COM
 B adj (AB) = |B| adj A ( A–1 = | A | adj A)

 B–1 B adj (AB) = |B| B–1 adj A


 adj (AB) = (adjB) (adj A)

Self practice problems :

(9) If A is non-singular, show that adj (adj A) = |A|n – 2 A.


(10) Prove that adj (A–1) = (adj A)–1.
( n 1) 2
(11) For any square matrix A, show that |adj (adj A) | = | A | .
(12) If A and B are non-singular matrices, show that (AB) –1 = B–1 A–1.

Elementary row transformation of matrix :


The following operations on a matrix are called as elementary row transformations.
(a) Interchanging two rows.
(b) Multiplications of all the elements of row by a nonzero scalar.
(c) Addition of constant multiple of a row to another row.
Note : Similar to above we have elementary column transformations also.
Remarks : Two matrices A & B are said to be equivalent if one is obtained from other using elementary
transformations. We write A  B.
Finding inverse using Elementry operations
(i) Using row transformations :
If A is a matrix such that A–1 exists, then to find A–1 using elementary row operations,
Step I : Write A = IA and
Step II : Apply a sequence of row operation on A = IA till we get, I = BA.
The matrix B will be inverse of A.
Note : In order to apply a sequence of elementary row operations on the matrix equation X = AB, we will
apply these row operatdions simultaneously on X and on the first matrix A of the product AB on RHS.

14
(ii) Using column transformations :
If A is a matrix such that A–1 exists, then to find A–1 using elementary column operations,
Step I : Write A = AI and
Step II : Apply a sequence of column operations on A = AI till we get, I = AB.
The matrix B will be inverse of A.
Note : In order to apply a sequence of elementary column operations on the matrix equation X = AB, we
will apply these row operatdions simultaneously on X and on the second matrix B of the product AB on
RHS.
0 1 2
 
1 2 3
Example # 15 : Obtain the inverse of the matrix A =  using elementary operations.
3 1 1

0 1 2 1 0 0
   
1 2 3 0 1 0
Solution : Write A = IA, i.e.,  =  A
3 1 1 0 0 1

1 2 3 0 1 0 
   
 0 1 2  =  1 0 0  A (applying R1  R2)
or
3 1 1 0 0 1

1 2 3  0 1 0
   
or 0 1 2 
= 
1 0 0
A (applying R3  R3 – 3R1)
0 – 5 – 8 0 – 3 1

1 0 – 1  – 2 1 0
   
or 0 1 2 
= 
1 0 0
A (applying R1  R1 – 2R2)

STUDYPIVOT.COM
0 – 5 – 8

 1 0 – 1
 0 – 3 1

 – 2 1 0
   
or 0 1 2  =  1 0 0
A (applying R 3  R3 + 5R2)
0 0 2   5 – 3 1

 
 1 0 – 1 – 2 1 0
   
or 0 1 2 =  1 0 0  A (applying R  1 R )
0 0 1   3
2 3
 5 –3 1
 2 2 2 

1 1 1
2 –
1 0 0 2 2
 
 
or  0 1 2  =  1 0 0
 A (Applying R1  R1 + R3)
0 0 1 5 3 1
2 –
 2 2 

 1 –1 1
1 0 0  2 2 2
   
or 0 1 0  =  – 4 3 – 1 A (Applying R  R – 2R )
0 0 1   2 2 3
 5 –3 1
 2 2 2 

15
 1 1 1
 2 –2 2
 
Hence A–1 = – 4 3 – 1
 
 5 3 1
 2 –2 2 

System of linear equations & matrices : Consider the system


a11 x 1 + a12x 2 + .......... + a1nx n = b1
a21x 1 + a22 x2 + ..........+ a2n xn = b2
.................................................
am1x 1 + am2x 2 + ..........+ amnx n = bn.

 b1 
 a11 a12 .......... a1n   x1   

a 21 a 22 .......... a 2n 
  
x b 2 
A =  , X =   & B =  ...  .
2
Let
..... ..... .......... .....  ....   
     ... 
a m1 a m2 .......... a mn  xn  b 
 n
Then the above system can be expressed in the matrix form as AX = B.
The system is said to be consistent if it has atleast one solution.

System of linear equations and matrix inverse:


If the above system consist of n equations in n unknowns, then we have AX = B where A is a square
matrix.

Results : (1) If A is non-singular, solution is given by X = A–1B.

STUDYPIVOT.COM
(2) If A is singular, (adj A) B = 0 and all the columns of A are not proportional, then the
system has infinitely many solutions.
(3) If A is singular and (adj A) B  0, then the system has no solution
(we say it is inconsistent).
Homogeneous system and matrix inverse :
If the above system is homogeneous, n equations in n unknowns, then in the matrix form it is AX = O.
( in this case b1 = b2 = ....... bn = 0), where A is a square matrix.

Results : (1) If A is non-singular, the system has only the trivial solution (zero solution) X = 0
(2) If A is singular, then the system has infinitely many solutions (including the trivial
solution) and hence it has non-trivial solutions.
xyz6
Example # 16 : Solve the system x  y  z  2 using matrix inverse.
2x  y  z  1

1 1 1  x 6 
     
Solution : Let A = 1  1 1  , X =  y  & B = 2 .
2 1  1  z   1
Then the system is AX = B.
|A| = 6. Hence A is non singular.

0 3 3 
 
Cofactor A = 2  3 1 
2 0  2

0 2 2
 
adj A = 3  3 0 
3 1  2
16
0 2 2  0 1/ 3 1/ 3 
1 1    
A–1 = | A | adj A =  3  3 0  = 1/ 2  1/ 2 0 
6
3 1  2 1/ 2 1/ 6  1/ 3

 0 1/ 3 1/ 3  6  x 1
       
X = A–1 B = 1/ 2  1/ 2 0 
 2 i.e.  y  = 2
1/ 2 1/ 6  1/ 3  1  z  3
 x = 1, y = 2, z = 3.
Self practice problems:

0 1 2
 
(13) A =  1 2 3  . Find the inverse of A using |A| and adj A.
3 1 1

(14) Find real values of  and µ so that the following systems has
(i) unique solution (ii) infinitely many solutions (iii) No solution.
x+y+z=6
x + 2y + 3z = 1
x + 2y + z = µ

(15) Find  so that the following homogeneous system have a non zero solution
x + 2y + 3z = x
3x + y + 2z = y
2x + 3y + z = z

STUDYPIVOT.COM
Answers : (13) 
1
2
1
2
4

3 
5
2
3
2
(14) (i)  3, µ  R (ii)  = 3, µ = 1 (iii)  = 3, µ  1 (15)  = 6
1 1
1
2 2

17
EXTRA TOPICS ( NOT FOR BOARDS)
Multiplication Of Two Determinants:
a1 b1 1 m1 a1 1b1 2 a1m1b1m 2
 
a2 b2 2 m2 a 2  1b 2  2 a 2 m1b 2 m 2

Summation of Determinants

n n n

f(r) g(r ) h(r ) n  f (r )  g(r )  h(r )


 (r ) =
r 1 r 1 r 1
Let (r) = a1 a 2 a 3 , then a1 a2 a3
b1 b 2 b 3 r 1
b1 b2 b3
where a1, a2, a3, b1, b2, b3 are constants indepedent of r

Integration of a determinant

b b b

f ( x ) g( x ) h( x )
b

a
f ( x ) dx 
a
g( x ) dx  h( x) dx
a
a1 b1 c1
Let (x) =
a2 b2 c2
, then  ( x ) dx = a1 b1 c1
a a2 b2 c2

STUDYPIVOT.COM
where a1, b1, c1, a2, b2, c2 are constants independent of x.

Differentiation of Determinant:
f1( x ) f2 ( x ) f3 ( x )
Let (x) = g1( x ) g2 ( x ) g3 ( x )
h1( x ) h 2 ( x ) h3 ( x )

f1( x ) f2 ( x ) f3 ( x ) f1( x ) f2 ( x ) f3 ( x ) f1( x ) f2 ( x ) f3 ( x )


then (x) = g1( x ) g2 ( x ) g3 ( x ) + g1 ( x ) g2 ( x ) g3 ( x ) + g1( x ) g2 ( x ) g3 ( x )
h1( x ) h2 ( x ) h3 ( x ) h1( x ) h 2 ( x ) h3 ( x ) h1 ( x ) h2 ( x ) h3 ( x )

Cramer's Rule: System of Linear Equations


(i) Two Variables
(a) Consistent Equations: Definite & unique solution. [ intersecting lines ]
(b) Inconsistent Equation: No solution. [ Parallel line ]
(c) Dependent equation: Infinite solutions. [ Identical lines ]

Let a1x + b1y + c 1 = 0 & a 2x + b 2y + c 2 = 0 then:


a1 b c
 1  1  Given equations are inconsistent &
a2 b2 c2
a1 b c
 1  1  Given equations are dependent
a2 b2 c2
18
(ii) Three Variables

Let, a1x + b1y + c 1z = d1............ (I)


a2x + b2y + c2z = d2............ (II)
a3x + b3y + c3z = d3............ (III)

D1 D2 D3
Then, x= ,Y= ,Z= .
D D D

a1 b1 c1 d1 b1 c1 a1 d1 c1 a1 b1 d1
a2 b2 c2 d2 b2 c2 a2 d2 c2 a2 b2 d2
Where D = ; D1 = ; D2 = & D3 =
a3 b3 c3 d3 b3 c3 a3 d3 c3 a3 b3 d3

(iii) Consistency of a system of Equations

(a) If D  0 and alteast one of D1, D 2, D3  0, then the given system of equations are consistent
and have unique non trivial solution.

(b) If D  0 & D 1 = D2 = D3 = 0, then the given system of equations are consistent and have trivial
solution only.

(c) If D = D 1 = D2 = D3 = 0, then the given system of equations have either infinite solutions or no
solution.

STUDYPIVOT.COM
(Refer Example & Self Practice Problem with*)
(d) If D = 0 but atleast one of D1, D2, D3 is not zero then the equations are inconsistent and have
no solution.
(e) If a given system of linear equations have Only Zero Solution for all its variables then the given
equations are said to have TRIVIAL SOLUTION.

(iv) Three equation in two variables :


If x and y are not zero, then condition for a1x + b1y + c 1 = 0 ; a2x + b2y + c 2 = 0 &

a1 b1 c1
a3x + b3y + c3 = 0 to be consistent in x and y is a 2 b2 c 2 = 0.
a3 b3 c3

19
EXERCISE - 1: IMPORTANT BOARD LEVEL QUESTIONS

Type (I) : Very Short Answer Type Questions : [01 Mark Each]

1. If a matrix has 5 elements, write all possible orders it can have.

x  2y 3 y 0 – 3
2.   =   , find the values of x and y..
 4 x 2  8 2 

 1 2  3 – 1
3. If A =   and B =   find AB.
2 3  – 1 0 

 2 1
   1 0 1
3 2
4. If A =  and B =   , find AB.
 1 1  1 2 1

 a b  2  5 
5. Find the value of a and b for which the following holds :     =  
 – a 2b   – 1 4

3 4
6. If A =   , find A + A, where A is the transpose of matrix A.
2 3

2 3

STUDYPIVOT.COM
7. If A =   , prove that A – AT is a skew symmetric matrix, where AT denotes the transpose of A.
4 5

1  2
 2 1
8. Evaluate : 2
 1 

a  ib c  id
9. Evaluate :
– c  id a – ib

2 cos  – 2 sin 
10. Evaluate :
sin  cos 

2 –3 5
11. Find the co-factor of a12 in the following : 6 0 4 .
1 5 –7

12. A matrix A of order 3 × 3 is such that |A| = 4. Find the value of |2A|.

Type (II) : Short Answer Type Questions : [02 Marks Each]

1
13. Construct a 2 × 2 matrix whose elements are given by aij. = | i – 3j |.
2

20
(i  2 j)2
14. Construct a 2 × 3 matrix A, whose elements are given by aij = .
2

 1  3 2 2  1  1
15. If A =  ,B=   , find the matrix C such that A + B + C is a zero matrix.
 2 0 2 1 0  1

 1 2 3   1
   
4 5 6  – 2
16. Find the value of x, if [1 x 1]  =0
3 2 5   3 

 – 1 2 3 – 2
17. Find the matrix X such that 2A + B + X = O, where A =   and B =  .
 3 4 1 5

0 0 1
 
0 1 0
18. If A =  , verify that A2 = I.
 1 0 0

 3  4
19. If A =   , find a matrix B such that AB = .
 1 2 

3
 
5
20. If A =   and B = [1 0 4] find (AB).
2

2 – 1  0 4
21. If A = 
3 2
STUDYPIVOT.COM
,B=   , find 3A2 – 2B + I.
 – 1 7

2( x  1) 2x 
22. For what value of x is A =   a singular matrix?
 x x – 2

2 3 
23. If A =   , write A in terms of A.
–1

5 – 2
0 ab 2 ac 2
2
a b 0 bc 2
24. Using the properties of determinants, evaluate the following : .
a 2c cb 2 0

xy x x
25. Using properties of determinants prove the following : 5 x  4 y 4 x 2 x = x3
10 x  8 y 8 x 3 x
Type (III) : Long Answer Type Questions: [04 Mark Each]

26. Using matrices, solve the following system of equations


x+y+z=6
x–y+z=2
2x + y – z = 1

3 1
27. If A =   , find x and y such that A2 + xI = yA. Hence find A–1.
7 5

21
 1 3 5
 
28. Express the matrix A =  – 6 8 3  as the sum of a symmetric and a skew-symmetric matrix.
 – 4 6 5 

3 4
29. Compute the adjoint of the matrix A =   and verify that (Adj.A) . A = | A | I.
5 7 

ax y z
x ay z
30. Using the properties of determinants, show that = a2 (a + x + y + z).
x y az

bc c a ab
c a ab bc
31. Using properties of determinants, prove that = 2(a + b + c) (ab + bc – a2 – b2 – c2).
ab bc c a

x 1 x  2 x  a
x2 x3 xb
32. Show that = 0, where a, b, c are in A.P..
x3 x4 xc

bc a b
ca c a
33. Using properties of determinants, show that = (a + b + c) (a – c)2
ab b c

x–2 2x – 3 3x – 4
34.
STUDYPIVOT.COM
Using properties of determinants, solve the following for x : x–4 2x – 9
3 x – 16 = 0
x – 8 2x – 27 3 x – 64

Type (IV) : Very Long Answer Type Questions: [06 Mark Each]

35. Using matrix method solve the following system of linear equations :
2x – 3y + 5z = 11
3x + 2y – 4z = –5
x + y – 2z = –3
36. Using matrix method solve the following system of linear equations :
x+y+z=1
x – 2y + 3z = 2
x – 3y + 5z = 3

 8 – 4 1
 
 10 0 6 
37. If A = , find A–1.
8 1 6 
 
Hence solve the following system of equations :
8x + 4y + z = 5
10x + 6z = 4
8x + y + 6z = 5/2

 1 3 – 2
 
– 3 0 – 1
38. Using elementary transformations, find the inverse of the matrix. 
 2 1 0 
 

22
 1 – 2 0  7 2 – 6
   
2 1 3  – 2 1 – 3 , find AB Hence solve the system of equations :
39. If A =  and B =
0 – 2 1  – 4 2 5 
x – 2y = 10, 2x + y + 3z = 8 and –2y + z = 7
 1 2 – 2
 
 –1 3 0 
40. Find the inverse of the following matrix using elementry operations : A =
 0 –2 1 
 
 1 – 1 0  2 2 – 4
   
 2 3 4  – 4 2 – 4
41. If A = and B = , find AB. Use this to solve the following system of equations :
0 1 2  2 –1 5 
   
x–y=3
2x + 3y + 4z = 17
y + 2z = 7
42. Gaurav purchases 3 pens, 2 bags and 1 instrument box and pays Rs. 41. From the same shop Dheeraj
purchases 2 pens, 1 bag and 2 instrument boxes and pays Rs. 29, while Ankur purchases 2 pens, 2 bags
and 2 instrument boxes and pays Rs. 44. Translate the problem into a system of equations. Solve the
s yst em of equations by m atrix m etho d and henc e f ind the cos t of 1 pen, 1 bag an d
1 instrument box.

a  b  2c a b
43. Using properties of determinants, prove the following : c b  c  2a b = 2(a + b + c)3.
c a c  a  2b

a b c
44. Using properties to determinants, prove the following : a – b b – c c – a = a3 + b3 + c3 – 3abc
bc c a ab

STUDYPIVOT.COM (b  c ) 2 ab ca
ab (a  c ) 2 bc
45. Using properties of determinants, show the following : = 2abc (a + b c)3.
ac bc (a  b )2

23
EXERCISE - 2: CBSE PREVIOUS YEARS QUESTIONS
Matrices

1. Find the values of a, b, c and d from the following equations.


2a  b a  2b   4  3
  =  . [CBSE 2004, 2005, 2008, 2009, 2010]
5c  d 4c  3d 11 24 

 3  5
2. If A =   , show that A2 – 5A – 14 = 0. [CBSE 2004, 2005, 2006, 2007, 2008, 2009, 2010]
 4 2 

3. Using matrix method solve the following system of linear equations :


x+y+z=3
2x – y + z = 2
x – 2y + 3z = 2 [CBSE 2004, 2005, 2006, 2007, 2008, 2009, 2010, 2011]

 cos A sin A   cos nA sin nA 


4. If X =   Prove that Xn =  ,nN [CBSE 2004, 2005, 2006]
 sin A cos A   sin nA cos nA 
 – 2
 
5. If A =  4  and B = [1 3 – 6]. Find B'A'. [CBSE 2005, 2007]
 5 

2 – 3 5 
 
3 2 – 4
6. If A =  , find A–1. Use it to solve the following system of equations :
1 1 – 2
2x – 3y + 5z = 16
3x + 2y – 4z = – 4

7.
2 4 
If A = 
STUDYPIVOT.COM
x + y – 2z = – 3
 1 3
,B= 
– 2 5
 and C =   Find the value of 2A – B – 3C.
[CBSE 2005, 2006, 2010, 2011]

[CBSE 2005, 2006]


3 2  – 2 5  3 4

 1 3 5
 
– 6 8 3
8. Express the following matrix as the sum of a symmetric and a skew symmetric matrix  .
 – 4 6 5 
[CBSE 2006, 2008, 2009]

0 1 1
  A 2  3
1 0 1
9. Find A–1 if A =  . Also show that A–1 = . [CBSE 2006]
 1 1 0 2

10. Given

1 – 1 1  – 4 4 4
   
1 – 2 – 2 – 7 1 3
A=  ,B=
2 1 3   5 – 3 – 1
Find AB and use this result in solving the following system of equations [CBSE 2006, 2010]
x–y+z=4
x – 2y – 2z = 9
2x + y + 3z = 1

24
(i  j )2
11. Construct a 2 × 2 matrix, A = [ aij ], whose elements are given by aij =
2
[CBSE 2007, 2008, 2011]

12. For what value of x, is the following matrix singular ? [CBSE 2008, 2011]
3 – 2x x  1
 
 2 4 

3 0 – 1
 
2 3 0
13. Obtain the inverse of the following matrix using elementary operations; A = 
0 4 1
[CBSE 2009, 2008, 2011]

14. If matrix A = (1 2 3), write AA, where Ais the transpose of matrix A. [CBSE 2009]

 cos  – sin  
15. If A =   , then for that value of  is A an identity matrix ? [CBSE 2010]
 sin  cos  

 2 – 1
16. Write the adjoint of the following matrix :   [CBSE 2010]
4 3 
17. For the following matrices A and B verify that (AB) = BA. [CBSE 2010]

 1 
 
 – 4
A= , B = (– 1 2 1).
 3 
 

18.
STUDYPIVOT.COM
If A is a matrix of order 3 × 4 and B is matrix of order 4 × 3, find the order of the matrix (AB).
[CBSE 2010]

Determinant
1 bc a(b  c )
1 ca b(c  a)
19. Using the property of determinants, prove that = 0. [CBSE 2004]
1 ab c(a  b)
20. Using properties of determinants, solve for x :
ax ax ax
ax ax ax
=0 [CBSE 2004, 2005,
ax ax ax
2009]

21. Using properties of determinants, show that :

1 a 1 1
1 1 b 1
= abc + ab + bc + ca. [CBSE 2004]
1 1 1 c

22. Using properties of determinants, prove that :

25
abc 2a 2a
2b bc a 2b
= (a + b + c)3 [CBSE 2004]
2c 2c c ab

1 1 1
23. Using properties of determinants prove that a b c = (a – b) (b – c) (c – a) (a + b + c)
a3 b3 c3
[CBSE 2005]

24. Using properties of determinants, prove the following : [CBSE 2006]

ab bc c a a b c
bc c a ab b c a
=2
c a ab bc c a b

sin  cos  cos(  )


sin  cos  cos(  )
25. Without expanding show that =0 [CBSE 2007]
sin  cos  cos(   )

x4 2x 2x
2x x4 2x
26. Using the properties of determinants, prove the = (5x + 4) (4 – x)2.
2x 2x x4
[CBSE 2009, 2007]

x x2 yz

27.
STUDYPIVOT.COM
Using properties of determinants prove the following :
y y2
z z2
zx
xy
= (x – y) (y – z) (z – x) (xy + yz + zx)

[CBSE 2007, 2008]

28. Using properties of determinants prove the following :

a2  1 ab ac
2
ab b 1 bc
2
= 1 + a2 + b2 + c 2 . [CBSE 2008, 2011]
ca cb c 1

29. Using properties of determinants, prove the following :

1  a2 – b2 2ab – 2b
2 2
2ab 1– a  b 2a
= (1 + a2 + b2)3 [CBSE 2008]
2b – 2a 1 – a2 – b2

30. Using properties of determinants, prove the following : [CBSE 2009]

1 1 p 1 p  q
2 3  2p 1  3p  2q
=1
3 6  3p 1  6p  3q

26
31. Find the minor of the element of second row, and third column (a23) in the following determinent :
2 –3 5
6 0 4
[CBSE 2010]
1 5 –7

32. If A is a square matrix of order 3 and |3A| = K|A|, then write the value of K. [CBSE 2010]

33. A is a square matrix of order 3 and |A| = 7. Write the value of |adj . A|. [CBSE 2010]

34. If a, b, c are positive and unequal, show that the following determinant is negative :
a b c
b c a
= [CBSE 2010]
c a b

35. Using matrix method, solve the following system of equation : [CBSE 2011]
2 3 10 4 6 5 6 9 20
  = 4, –  = 1,  – = 2 ; x, y, z  0
x y z x y z x y z

STUDYPIVOT.COM

27
SOLUTIONS
EXERCISE: 1 1  2
 2 1
1. Since a matrix of order m × n has mn elements, 8. We have given, 2
therefore, to find all possible orders of a matrix with 5  1 
elements, we have to find all possible ordered pairs Apply C1  C1 + C2 + C3
(m, n) of positive integers whose product is 5. Hence
possible orders are 1 × 5 and 5 × 1. 1    2  2 0  2
2 2 2
  1  1 0  1
x  2y 3 y 0 – 3 = 2
= =0
2.   =     1  1  0 1 
 4x 2 8 2 
By the properties of equality of matrices, we have [ 1 +  + 2 = 0 where  is the cube root of unity]
4x = 8 and 3y = – 3  x = 2 and y = –1
a  ib c  id
 1 2  3 – 1 9.
– c  id a – ib
3. AB =   
2 3   – 1 0  = (a + ib) (a – ib) – (– c + id) (c + id)
= (a2 – i2b2) – (i2d2 – c2)
3 – 2 – 1  0   1 – 1 = (a2 + b2) – (– d2 – c2) [ i2 = – 1]
AB =   =  
6 – 3 – 2  0  3 – 2 =a +b +d +c
2 2 2 2

= a2 + b2 + c2 + d2
 2 1
 
3 2  1 0 1 2 cos  – 2 sin 
4. AB =    10. = 2cos2 + 2sin2 = 2(cos2 +
 1 1  1 2 1 sin  cos 
sin2) = 2.1 = 2
 2 1 0  2 2 1   1 2 3
    6 4
32 04 32 1 4 5
=  =  11. Cofactor of a12 = (–1)3 = – (– 42 – 4) = 46
 1  1 0  2  1  1  2 2 0 1 –7

5. Here given, 
STUDYPIVOT.COM
 2a – b  5
 =  
12. A is a 3 × 3 matrix s.t. |A| = 4.
If A = (aij)3×3 , then 2A = (2aij)3×3
 |2A| = 23 |A| = 8 × 4 = 32.
 – 2a – 2b 4
By definition of equality of matrices
a11 a12 
2a – b = 5 13. Let A = a  be a 2 × 2 matrix
– 2a – 2b = 4  21 a 22 
On adding (i) and (ii), we get
–3b = 9  b = – 3 1
 a11 = |1–3|=1
Putting b = – 3 in equation (i), we get 2
2a + 3 = 5 a = 1 1 5
a12 = |1–6|=
2 4  3 2  2 2
6. We have A =    A =  
2 3  4 3 1 1
a21 = |2–3|=
2 4  3 2  6 6  2 2
 A + A =   +   =   1
 2 3   4 3  6 6  a22 = |2–6|=2
2
2 3 2 4 
7. A =    AT =    5
4 5 3 5  1 2
Hence the required matrix is A =  1 
2 3 2 4  0  1  2
Let B = A – AT =   –   =   2 
 4 5  3 5  1 0  2
(i  2 j)
14. aij = where 1  i  2; 1  j  3
 0 1 0  1 2
 BT =   = –  = –B
 1 0 1 0  (1  2)2 1 (1  4)2 9
 B = A – AT is a skew symmetric matrix.  a11 = = ; a12 = = ;
2 2 2 2
28
(1  6)2 25 ( 2  2 )2  3  4
a13 = = ; a21 = = 0; 19. Given : A =  
2 2 2  1 2 
( 2  4)2 ( 2  6) 2 a b
a22 = = 2; a23 = =8 Let B =   s.t. AB = 
2 2  c d
 The reqd. 2 × 3 matrix is
 3  4 a b  1 0
1 9 25       =  
a11 a12 a13     1 2   c d 0 1
A = a  = 2 2 2 
 21 a 22 a 23 
0 2 8   3a – 4c = 1 ; 3b – 4d = 0
–a + 2c = 0 ; – b + 2d = 1
1  3 2 2  1  1 Solving these, we get
15. Here, A =   and B =  
2 0 2 1 0  1 a = 1; c =
1
; b = 2, d =
3
we want to find C s.t. 2 2
A+B+C=0
a b 1 2
 C =O –A– B=–A–B
 = 
1 3
 B= 
 1  3 2 2  1  1  c d 2 2 
=–   –  
 2 0 2 1 0  1
3 3 0 12 
  1 3  2  2 1 1  3 4  1    
=   +   =   5 5 0 20
20. Here AB =   [1 0 4] =
  2 0  2   1 0 1  3 0  1 2 2 0 8 

 1 2 3  1 3 5 2
   
4 5 6  – 2 0 0 0
16. [1 x 1]  =0  (AB) =
3 2 5  3 12 20 8

6
  2 – 1
12 21. A =  
 [1 x 1]   = 0  [6 + 12x + 14] = 0 3 2
14 

 12x + 20 = 0
STUDYPIVOT.COM
 x=–
5
2 – 1 2 – 1
 A2 = 
3 2
 
 3 2 
 1 – 4
 = 
12

1
3
 3A2 – 2B + 
17. Here 2A + B + X = O
 1 – 4  0 4  1 0
 – 1 2  3 – 2  0 0 = 3  – 2  +  
12 1  – 1 7 0 1
2   +X=  
 3 4  1 5 0 0
 3 – 12  0 8  1 0
– 2 4 3 – 2 0 0 =   –   +  
36 3   – 2 14  0 1
 +   +X=  
 6 8 1 5  0 0
 3 – 0  1 – 12 – 8  0  4 – 20
1 2  0 0 1 2  =   =  
36  2  0 3 – 14  1  38 – 10 
  +X=   X=–  
7 13 0 0 7 13
22. For the given matrix,

0 0 1 0 0 1 2( x  1) 2x 
    A=  
0 1 0  0 1 0   x x – 2
18. A2 = A · A = 
 1 0 0   1 0 0  to be singular, |A| = 0
2( x  1) 2x
 0  0  1 0  0  0 0  0  0 1 0 0 0= = 2(x + 1) (x – 2) – 2x2
x x–2
   
0  0  0 0  1  0 0  0  0 0 1 0
=  =   x = –2
0  0  0 0  0  0 1  0  0  0 0 1
23. Here | A | = 2(–2) – 5(3)
 A2 =  = 4 – 15 = – 19  0

29
C11 = (– 1)2 (1 – 1) = 0
1
 A–1 = · adj A C12 = (– 1)3 (–1 – 2) = 3
|A| C13 = (– 1)4 (1 + 2) = 3
C21 = (– 1)3 (–1 – 1) = 2
1  – 2 – 3 1 2 3 
= – =   C22 = (– 1)4 (–1 – 2) = – 3

19  – 5 2  19 5 – 2 C23 = (– 1)5 (1 – 2) = 1
C31 = (– 1)4 (1 + 1) = 2
0 ab 2 ac 2 C32 = (– 1)5 (1 – 1) = 0
C33 = (– 1)6 (–1 – 1) = – 2
a 2b 0 bc 2
24. Let  =
a 2c cb 2 0 0 2 1 
 
3 –3 0 
Take : a2, b2 and c2 respectively common from adj. A = 
 3 1 – 2
C1, C2 and C3.  

0 a a 0 2 1 
1 1  
b 0 b  A–1 = adj. A = 3 – 3 0 
 = a2 b2 c2 |A|
c c 0 6  
 3 1 – 2
operate : C2  C2 – C3 X = A–1B
0 0 a x 0 2 2  6
     
b b b  y = 1 3 – 3 0   2
 = a2 b2 c2
c c 0 z 6  3 1 – 2  1
     
Expand by R1
x  042  6
b b   1  1 
 = a b c .a =
2 2 2
= 2a b c .
3 3 3 y
  =  18 – 6  0  = 12 
c c z 6  6 
   18  2 – 2  12 
xy x x
6
25. Let  = 5 x  4 y 4 x 2 x  x= =1
6

STUDYPIVOT.COM
10 x  8y 8x 3 x
12
Operate : R2  R2 – 2R1; R3  R3 – 3R1 y= =2
6
xy x x 18
z= =3
3x  2y 2x 0 6
=
7x  5 y 5 x 0  x = 1, y = 2, z = 3
Expand by C3
3 1
3 x  2y 2x 27. A =  
=x 7 5
7x  5 y 5 x
3 1
= x[5x(3x + 2y) – 2x(7x + 5y)]  A2 =  
= x[15x2 + 10xy – (14x2 + 10xy)] = x3 7 5
Now A2 + xI = yA ...(1)
26. The given equations can be written as
AX = B  X = A–1B 16 8   1 0 3 1
   + x  = y 
56 32  0 1 7 5
1 1 1 x 6
     
 1 – 1 1  y  2
A= ,X =  , B= 16  x 8  3 1
 2 1 – 1 z  1    =y  
       56 32  x  7 5
 16 + x = 3y ; 8=y
1 1 1 56 = 7y ; 32 + x = 5y
1 –1 1 Solving these, we get
|A|=
2 1 –1 y = 8, x = 8
As | A | = 15 – 7 = 8  0 A–1 exists
= 1(1 – 1) – 1(– 1 – 2) + 1 (1 + 2) Multiplying (1) by A–1, we get
=3+3=60 (A2 + x I) by A–1, we get
A–1 exists (A2 + x I) A–1 = yAA–1

30
 A + 8A–1 = 8I ( x = y = 8)
a a 0
 1 0 3 1  5 – 1 x ay z
8A–1 = 8   –   =   =
0 1 7 5  – 7 3
x y az
1  5 – 1 Operate : C2  C2 + C1
 A–1 =  
8 – 7 3
a 0 0
28. We know that x ayx z
=
1 1 x yx az
A= (A + A') + (A – A')
2 2 Expand by R1
1 ayx z
Here (A + A') is a symmetric matrix and = a
2 yx az
1 = a[(a + y + x) (a + z) – (y + x)z] = a[a2 + az + (y +
(A – A') is a skew-symmetric matrix.
2 x) a + (y + x)z – (y + x)z] = a2 (a + x + y + z).

 1 3 5 1 – 6 – 4 bc c a ab
    c a ab bc
– 6 8 3 3 8 6  31. Let  =
Now A=   A' = 
 – 4 6 5  5 3 5  ab bc c a
Operate : C1  C1 + C2 + C3
1
 (A + A') 2(a  b  c ) c  a a  b
2
2(a  b  a) a  b b  c
 1 3 5  1 – 6 – 4    = = 2(a + b + c)
2(a  b  c ) b  c c  a
1  – 6 8 3  3 8

6 
=   
2  – 4 6 5 5 3 5  1 c a ab
  
1 ab bc
 2 –3 1 1 bc ca
1  Operate : R2  R2 – R1 ; R3  R3 – R1
– 3 16 9 

STUDYPIVOT.COM
= 2 which is a sym. metrix
 1 9 10  1 c a ab
0 bc c a
0 9 9   D = 2(a + b + c)
0 ba c b
1 1  – 9 0 – 3
and (A – A') =  
2 2 – 9 3 0  bc c a
 
= 2(a + b + c) 1. open w.r.t. R1
Which is skew-sym. metrix ba c b
= 2(a + b + c) [(b – c) (c – b) – (c – a) (b – a)]
3 4 = 2(a + b + c) [bc – b2 – c2 + cb – (cb – ac – ab + a2)]
29. Here, A =    | A | = 21 – 20 = 1. = 2(a + b + c) (ab + bc + ca – a2 – b2 – c2)
5 7 
32.  a, b, c are in A.P.
 7 – 4 b–a=c–b ...(i)
Adj. A =  
– 5 3
x 1 x  2 x  a
 7 – 4 3 4 x2 x3 xb
Now (Adj.A) . A =  .   Now L.H.S. =
– 5 3 5 7  x3 x4 xc

 1 0  1 0 Operate : R1  R2 – R1 ; R3  R3 – R2
=   = 1.  
0 1 0 1 x 1 x  2 x  a
1 1 b–a
=
ax y z 1 1 c –b
x ay z
30. Let  =
x y az x 1 x  2 x  a
Operate : R1  R1 – R2 1 1 b–a
= [Using (1) ]
1 1 b–a

31
= 0 [ R2 and R3 are identical] 35. We can write the given eqs. as
= R.H.S. AX = B ...(1)

2  3 5  x
bc a b    
3 2  4 y ;
ca c a Where A =  ;X=
33. L.H.S. = 1 1  2  z 
ab b c
Operate : R1  R1 + R2 + R3  11 
 
2(a  b  c ) a  b  c a  b  c 5
B=  
ca c a  3
=
ab b c
2  3 5 
 
2 1 1 3 2  4
Now |A| = 
ca c a 1 1  2 
= (a + b + c)
ab b c = 2(–4 + 4) + 3(–6 + 4) + 5(3 – 2) = –6 + 5 = –1  0
Operate : C1  C1 – 2C3, C2  C2 – C3 1
 A–1 exists and it is given by A–1 = | A | .adj.A
0 0 1
c–a c–a a 0  1 2   0 1 2 
= (a + b + c)
a  b – 2c b – c c    
2  9 23  2 9  23
= (–1)  =
1  5 13    1 5  13 
1 1
= (a + b + c) (c – a)
a  b – 2c b – c
 0 1 2   11  x 1
[open w.r.t. R1]        
 2 9  23  5   y  = 2 
= (a + b + c) (c – a) (b – c – a – b + 2c) X = A–1 B = 
  1 5  13   3  z  3
= (a + b + c) (c – a) (c – a)
= (a + b + c) (c – a)2 x = 1, y = 2, z = 3
= (a + b + c) (a – c)2 = R.H.S which is the reqd. sol. of the given eqs.

34. LHS
x–2
x–4
STUDYPIVOT.COM
2x – 3
2x – 9
3x – 4
3 x – 16
36. The given equations can be written as
AX = B ...(1)

x – 8 2x – 27 3 x – 64 1 1 1 x 1
     
1  2 3 y ; B = 2 
Applying R1  R1 – R2 and R2  R2 – R3 where A =  ;X=
1  3 5   z  3
2 6 12
4 18 48 1 1 1
=
x – 8 2x – 27 3 x – 64 1 2 3
Now |A| =
1 3 5
1 3 6
= 1(–10 + 9) – 1(5 – 3) + 1(–3 + 2)
=2×2 2 9 24 = –1 – 2 – 1 = –4  0
x – 8 2x – 27 3 x – 64
1
Applying R2  R2 – 2R1  A–1 exists and it is given by A–1 = | A | adj.A

1 3 6
1  8 5  1 8  5
=4 0 3 12 1   1  
=–  2 4  2 = 2  4 2 
x – 8 2x – 27 3x – 64 4   1 4  3 4 1  4 3 
 
Expanding by C1
Now eq. (1) 
= 4[3(3x – 64) – 12(2x – 27) + (x – 8) (3 × 12 – 3 × 6)
= 4[9x – 192 – 24x + 324 + 18(x – 8)] 1 8  5  1
= 4[3x – 12] = 0 (Given) 1    
 x=4 X = A–1 B = 2  4 2  2 
4 1  4 3  3

32
x 2 1/ 2  1 3 – 2
       
1 1 1
 y = 0 =  0   x = , y = 0, z = 38. Consider A = 
– 3 0 – 1
 z  4 2 1/ 2 2 2  2 1 0 
   
This is the reqd. sol. of the given equations. We have A = IA

 1 3 – 2  1 0 0
 8 – 4 1    
   – 3 0 – 1 =  0 1 0  A
 10 0 6   2 1 0   0 0 1
37. Part I : We have A =
8 1 6     

[R2  R2 + 3R1 and R3  R3 – 2R1]
8 –4 1
 1 3 – 2  1 0 0 
10 0 6    
|A|= 0 9 – 7 =  3 1 0 A
8 1 6  0 – 5 4   – 2 0 1
   
= 8(0 – 6) + 4(60 – 48) + 1(10 – 0)
= – 48 + 48 + 10 = 10  0  R2 
 A–1 exists R 2  9 
 
A11 = + (0 – 6) = – 6
A12 = – (60 – 48) = – 12
1 3 – 2   1 0 0
A13 = + ()10 – 0 = 10   1 
A21 = – (– 24 – 1) = 25 0 1 – 7  1 0 A
 9 =  3 9 
A22 = + (48 – 8) = 40
0 – 5   – 2 0 1
A23 = – (8 + 32) = – 40 
4
  
A31 = + (– 24 – 0) = – 24
[R3  R3 + 5R2]
A32 = – (48 – 10) = – 38
A33 = + (0 + 40) = 40 1 3 – 2 
   1 0 0 
0 1 – 7  
 –6 25 – 24 
1 1    9 =  1 1 0 A
– 12 40 – 38   3 9 
 A–1 = | A | adj. A = 10  0 0 1 
–1 5 1
 10 – 40 40   9 
   3 9 

as
STUDYPIVOT.COM
Part 2 : Given system of equations can be written
[R3  9R3]

1 3 – 2 
 
 1

0 0

AX = B  X = A–1 B 0 1 – 7   13 1 0
9 = 9 A
where 0 0 1   – 3 5 9 
 
 8 –4 1 x  5  [R1  R1 – 3R2]
     
10 0 6 y  4 
A=  ,X =  ,B=
8 1 16  z 5 / 2 1 0 1   0 – 1/ 3 0 
       3   
 0 1 – 7  =  1/ 3 1/ 9 0  A
Using Part I, we have –3
 9
 5 9 
0 0 1 
 –6 25 – 24   
1  
– 12 40 – 38 
A–1 = 10   1 7 
 10 – 40 40 
  R1  R1 – 3 R 3 ,R 2  R 2  9 R 3 
 
x  –6 25 – 24   5   1 0 0  1 – 2 – 3
  1       
y  – 12 40 – 38   4 
   10  0 1 0 =  – 2 4 7 
A
 z   10 – 40 40  5 / 2  0 0 1 – 3 5
      9 

 – 30  100 – 60  10   1   1 – 2 – 3
1   1      
– 60  160 – 95  5   1/ 2
= 10  10  A =
–1  –2 4 7 
 50 – 160  100   – 10  – 1 – 3 5
 9 
1
 x = 1, y = , Z = –1
2

33
 1 – 2 0 1 2 – 2 1 0 0
     
2 1 3 – 1 3 0  0 1 0
39. Here , A =  =  A
0 – 2 1  0 – 2 1  0 0 1
Applying R2  R2 + R1, we get
 7 2 – 6
  1 2 – 2 1 0 0
– 2 1 – 3
B=     
 – 4 2 5  0 5 – 2 =  1 1 0 A
0 – 2 1  0 0 1
1 – 2 0  7 2 – 6
    1
2 1 3  – 2 1 – 3  Applying R2  R , we get
 AB =  · 5 2
0 – 2 1  – 4 2 5 
1 2 – 2 1 0 0
11 0 0  1 0 0  – 2 1 1 
    0 1  =  0
A
0 11 0 0 1 0  5  5 5 
=   = 11
1 = 11
1I3
 0 0 11 0 0 1 0 – 2 1  0 0 1
Applying R3  R3 + 2R2, we get
 1  1
 A ·  B  = 3  A–1 = B    
 11  11 1 2 – 2 1 0 0
Now the given system of equation is  – 2 1 1 
x – 2y = 10 0 1   0
 5  = 5 5 A
2x + y + 3z = 8 0 0 1  2 2
– 2y + z = 7    5 1
5  5 
1 – 2 0  x  10  Applying R3  5R3, we get
    
2 1 3  y  8
  =   1 2 – 2 1 0 0
0 – 2 1  z   7   – 2 1 1 
0 1  =  0
 5  5 5 

STUDYPIVOT.COM
x 10  0 0 1  2 2 5
   
y 8
A·   =   2
 z   7  Applying R2  R2 + R
5 3
and R1  R1 + 2R3, we get
x 10 
   
y 8 1 2 0 5 4 10
   = A–1   (From above A–1 exists)    
 z   7 
0 1 0 = 1 1 2  A
0 0 1 2 2 5 
10   7 2 – 6 10
1 Applying R1  R1 – 2R2, we get
1 8  
– 2 1 – 3  8 
= B   = 11 
11  7   – 4 2 5   7  1 0 0 3 2 6
     
0 1 0  =  1 1 2 A
 44   4  0 0 1 2 2 5 
1    
– 33   – 3
= 11  3 2 6
 11   1   
1 1 2
Hence A–1 = 
 x = 4, y = –3, z = 1 2 2 5 

1 – 1 0  2 2 – 4
1 2 – 2    
  2 3 4  – 4 2 – 4
–1 3 0  41. Part I : AB = 
40. Let A =  0 1 2  2 – 1 5 
 0 – 2 1 
In order to use elementary row transformation, we  240 2–2–0 – 440 
may write A = IA  
4 – 12  8 4  6 – 4 – 8 – 12  20
= 
 0 – 4  4 0  2 – 2 0 – 4  10 

34
6 0 0 1 0 0 a  b  2c a b
   
0 6 0 0 1 0 c b  c  2a b
=  = 6 = 6I 43. LHS =
0 0 6 0 0 1 c a c  a  2b
Part II : Given system of equations can be written as Operate : C1  C1 + C2 + C3
AX = C  X = A–1C
2(a  b  c ) a b
1 – 1 0 x 3 2(a  b  c ) b  c  2a b
      =
2 3 4 y , C = 17  2(a  b  c ) a c  a  2b
where A =  ,X=
0 1 2  z   7 
Using part I, we have AB = 6I  A–1 AB = 6A–1I 1 a b
1 b  c  2a b
 2 2 – 4 = 2(a + b + c)
1 a c  a  2b
1 1 – 4 2 – 4

 B = 6A –1
 A–1 = B=  
6 6  2 –1 5  Operate : R2  R2 – R1 ; R3  R3 – R1
 
1 a b
x  2 2 – 4  3 
  1    0 (a  b  c ) 0
y – 4 2 – 4 17 = 2(a + b + c)
Now   =
6  0 a (c  a  b )
 z   2 – 1 5   7 
Expand by C1
 6  34 – 28   12  2 abc 0
1  1    = 2(a + b + c) .1.
– 12  34 – 28 –6 –1 0 abc
= 6 = 6  =  
 6 – 17  35   24   4  3
= 2(a + b + c) = RHS
 x = 2, y = –1, z = 4 a b c
42. Let x, y, z be costs of 1 pen, 1 beg and 1 box a–b b–c c –a
44. L.H.S. =
respectively. bc c a ab
3x + 2y + z = 41 ------- (1)
2x + y + 2z = 29 ------- (2) Operate : C1  C1 + C2 + C3
2x + 2y + 2z = 44
x + y + z = 22 STUDYPIVOT.COM
------- (3)
=
abc
0
b c
b–c c–a
3 2 1  x   41 2(a  b  c ) c  a a  b
     
 2 1 2    = 29
y
 1 1 4   z  22 1 b c
Matrix method 0 b–c c–a
= (a + b + c)
AX = B  X = A –1B 2 c a ab

 1 1 3  Operate : R3  R3 – 2R1
1  
 0 2  4 1 b c
A–1 =
2  1 1 1 
  0 b–c c–a
= (a + b + c)
0 c  a – 2b a  b – 2c
1 1  3  41
1     Expand by C1
A–1 B =  0  2 4  29
2  1 1 1  22 b–c c–a
= (a + b + c) · 1
c  a – 2b a  b – 2c
x 4 Operate : R2  R2 + 2R1
  1  
y = 30    x = 2, y = 15, z = 5 b–c c–a
 z  2 10 
  = (a + b + c)
a–c b–a
= (a + b + c) [(b – c) (b – a) – (a – c) (c – a)]
= (a + b + c) [(b2 – ab – bc + ac) + (a2 + c2 – 2ac)]
= (a + b + c) (a2 + b2 + c2 – ab – bc – ca)
= a3 + b3 + c3 – 3abc = R.H.S

35
(b  c ) 2 ab ca EXERCISE : 2
2
ab (a  c ) bc CBSE Problems (Hints & Solutions)
45.  =
ac bc (a  b )2
1. By equality of two matrices, we have
Applying R1  aR1, R2  bR2 and R3  cR3 2a + b = 4 ...(i)
we get a – 2b = –3 ...(ii)
5c – d = 11 ...(iii)
a(b  c )2 a 2b a 2c 4c + 3d = 24 ...(iv)
1 On solving the above equations, we get
= ab b(a  c )2 b 2c
abc 2 2 a = 1, b = 2, c = 3 and d = 4.
ac b c c(a  b ) 2

 3  5  3  5  29  25 
(b  c ) 2 a2 a2 2. A2 = A.A. =  .  =  
abc  4 2   4 2   20 24 
= b2 (a  c ) 2 b2
abc A2 – 5A – 14
c2 c2 (a  b ) 2
 29  25   3  5  1 0
Applying C2  C2 – C1, C3  C3 – C1, we get =   – 5  – 14  
  20 24    4 2  0 1
(b  c )2 a 2 – (b  c )2 a 2 – (b  c ) 2
 29  25   15  25  14 0 
b2 ( a  c )2 – b 2 0 =   –   –  
=   20 24    20 10   0 14 
c2 0 (a  b )2 – c 2
 29  15  14  25  25  0 0 0
(a  b  c ) (a  b  c ) =   =   =O
(b  c )2  20  20  0 24  10  14  0 0
(a – b – c ) (a – b – c )
2 (a  b  c )
b 0 3. We can write the given eqs. as
= (a  c – b ) AX = B ...(1)
(a  b  c )
c2 0 1 1 1 x 3
(a  b – c )
     
2 1 1  y ;B= 2
Where A =  ;X=
(b  c ) 2

STUDYPIVOT.COM
(a – b – c ) (a – b – c )  1  2 3   z   2 
 = (a + b + c) b2 (a  c – b ) 0
2
c 0 (a  b – c ) 1 1 1
2 1 1
Applying R1  R2 – (R2 + R3), we get Now |A| =
1 2 3
2bc – 2c – 2b
= 1(–3 + 2) – 1(6 – 1) + 1(–4 + 1)
b2 ac –b 0 = –1 – 5 – 3 = –9  0
 = (a + b + c) 2

c2 0 ab–c
1
 A–1 exists and it is given by A–1 = | A | .adj.
1 1
Applying C2  C2 + C , C  C3 + C1, we get
b 1 3 c
1  5 2 
1 1  
5 2 1
A–1 = | A | .adj. A =  
2bc 0 0 9  3 3  3 
 
2 b2
 = (a + b + c)2 b ac From eq. (1),
c
c2 1  5 2  3
c2 ab
b 1  5 2 1
  
X = A–1 B = –  2
9  3 3  3  2
Expanding along R1, we get  
 = (a + b + c)2 [2bc (a2 + ab + ac + bc – bc)]
= (a + b + c)2 [2bc(a2 + ab + ac)] x  9  1
= (a + b + c)2 · 2abc (a + b + c)   1   
y 
  = 1
9
= 2abc(a + b + c)3    =–
 z  9  9 1
 x = 1, y = 1, z = 1
which is the reqd. sol. of the given eqs.

36
By calculating various cofactors, we get
 cos nA sin nA  0 –1 2
4. Let P(n) : Xn =  ; n  N 1
 sin nA cos nA  A –1
= · adj. A = – 2 – 9 23
|A|
For n = 1 , 1 – 5 13
 cos 1. A sin1. A  Now the given equations are
P(1) : X1 =  
 sin1. A cos 1. A  2x – 3y + 5z = 16
3x + 2y – 4z = – 4
 cos A sin A  x + y – 2z = – 3
 X =  sin A cos A 
  2 – 3 5   x   16 
which is given      
  3 2 – 4 y
   =  – 4
Suppose : P(m) is true
1 1 – 2  z   – 3
 cos mA sin mA 
i.e. , Xm =   x
 sin mA cos mA   16 
   
Now when n = m + 1,  AX = B where X =  y  and B =  – 4
Xm+1 = Xm . X  z   – 3
 cos mA sin mA   cos A sin A   A–1 (AX) = A–1B
=  .   X = A–1B
 sin mA cos mA   sin A cos A 
 cos mA cos A  sin mA sin A cos mA sin A  sin mA cos A  x 0 – 1 2  16 
=        – 4
  sin mA cos A  cos mA sin A  sin mA sin A  cos mA cos A 
 y = – 2 – 9 23  
[ cosA cosB  sinA sinB = cos(A ± B)] and sinA  z   1 – 5 13  – 3
cosB ± cosA sinB = sin(A ± B)
 cos(mA  A ) sin(mA  A )   – 2 2 
=      
 sin(mA  A ) cos(mA  A ) = –  – 1 =  1
 – 3  3
 cos(m  1)A sin(m  1)A 
=    x = 2, y = 1, z = 3
 sin(m  1)A cos(m  1)A 
 P(m + 1) is true
STUDYPIVOT.COM
 By the principle of induction, P(n) is true  n  N.
7. 2A – B – 3C
2 4   1 3   – 2 5
= 2 –  – 3 
 1 3 2  – 2 5  3 4
 
5. A' = [–2 4 5] and B' =  3
 – 6 4 8   – 1 – 3  6 – 15 
=    
6 4  2 – 5  – 9 – 12
 1
   9 – 10 
3
B'A' =   [–2 4 5] =  
 – 6  – 1 – 13 
8. For any square matrix A,
– 2 4 5 1 1
  A= (A + A') + (A – A')
–6 12 15  2 2
= 
 12 – 24 – 30 
1
Here (A + A') is a symmetric matrix and
2
2 – 3 5 
  1
3 2 – 4
6. A =  (A – A') is skew-symmetric matrix.
1 1 – 2 2

 1 3 5  1 – 6 – 4
2 –3 5    
– 6 8 3 3 8 6
3 2 –4 Now A =  A' = 
 |A|=  – 4 6 5  5 3 5
1 1 –2
= 2(– 4 + 4) + 3 (– 6 + 4) + 5(3 – 2) 1
 (A + A')
=0–6+5=–10 2
 A–1 exists.
37
  1 3 5   1 – 6 – 4   1 1 1 
   
1   – 6 8 3   3 8 6  1 1 1 
=    =  = 2A–1 [By (1)]
2   – 4 6 5  5 3

5   1 1  1
  

 2 –3 1 A 2  3
 A–1 = .
1  – 3 16 9  2
=  
2  1 9 10 10. Here,

Which is the symmetric part of A. 1 – 1 1   – 4 4 4
   
1 1 – 2 – 2  – 7 1 3
Also (A – A') A.B = A =  ·
2 2 1 3   5 – 3 – 1

  1 3 5  1 – 6 – 4 
1    8 0 0
 – 6 8 3 – 3 8 6   
= 2   = 
0 8 0
= 8I
   5 
  – 4 6 5 5 3 0 0 8

 0 9 9 1  1
1
– 9 0 – 3
  A  B = I  A–1 = B
= 2 8  8
 – 9 3 0 Now the given equations. can be written as
Which is the skew-symmetric part of A. 1 – 1 1  x 4
     
0 1 1 1 – 2 – 2 X = C where X = y ; C = 9 
  2 1 3   z   1
1 0 1
9. Here , A = 
 1 1 0 1
 AX = C  X = A–1 C = BC
 |A| = –1 (0 – 1) + 1(1 – 0) = 2  0 8
 A–1 exists and it is given by x

STUDYPIVOT.COM
1   1
1
A–1 = | A | .adj.A = .adj A.   y  = BC
2  z  8

Now calculating the various co-factors, we get x – 4 4 4  4


  
1 –7 1   
 1 1 1   y =  3 9 
   z  8  5 – 3 – 1  1
1 1 1   
adj. A = 
 1 1  1
 24  3
1    
 – 16  =  – 2
 1 1 1  =
  8  – 8  – 1
1
 A–1 =  1 1 1  ...(1)
2  1 1  1  x = 3, y = – 2, z = – 1
 
Now A2 =
a11 a12  (i  j )2
0 1 1 0 1 1 2 1 1 11. Let A = a  where aij =
       21 a 22  2
1 0 1  1 0 1 =  1 2 1
A.A = 
 1 1 0  1 1 0  1 1 2 (1  1)2
 a11 = =2
2
2 1 1 1 0 0
    (1  2)2 9
1 2 1 0 1 0 a12 = =
 A2 – 3 =  –3  2 2
 1 1 2 0 0 1
(2  1)2 9
a21 = =
2 2

38
( 2  2 )2  1 
a22 = =8  1 0 0
2 1 0 – 3  3
  – 2 1 
2
 2 9 / 2 0 1  =  0 A
 Required matrix A =    9  9 3 
9 / 2 8   1  8 –4 
1
0 0 9  9
 3 
3 – 2x x  1 
12. Let A =  
 2 4  Applying R3  9R3, we get
 Matrix A is singular
 |A| = 0  1  1 
0 0
1 0 – 3  3
3 – 2x x  1  – 2 
2 1
=0 0 1  =  0 A
2 4  9  9 3 
(12 – 8x) – (2x + 2) = 0 0 0 1  8 – 12 9
   
12 – 8x – 2x – 2 = 0    
– 10x + 10 = 0
– 10x = – 10  x = 1 2 1
Applying R2  R2 – R and R1  R1 + R3
13. Since A = IA 9 3 3

3 0 – 1 1 0 0 1 0 0  3 –4 3
       
0 1 0 – 3 3 – 2 A
 2 3 0
= 
0 1 0
A   = 
0 4 1 0 0 1 0 0 1  8 – 12 9

1  3 –4 3
Applying R1  R , we get  
3 1 –3 3 – 2
Hence A–1 = 
 8 – 12 9
 1 1 
1 0 – 3  3 0 0
2 3  
0 =  0 1 0 A 1
   
0 4 1  0 0 1 2

STUDYPIVOT.COM
14. Here A = (1 2 3)  A =  
    3
Applying R2  R2 – 2R1, we get
1
 1  1   
2
1 0 – 3  0 0  AA = [1 2 3]   = [1 + 4 + 9] = [14]
3 3
 2   2 
0 3  = – 1 0 A
 3  3  15.  = 0°
0 4 1  0 0 1
   
     2 – 1
16. Let A =  
4 3 
1
Applying R2  R , we get
3 2  3 1
 adj A =  
 – 4 2
 1  1 
1 0 – 3  3 0 0
 – 2 
2 1  1  –1 2 1 
0 1  =  0 A
 9  9 3     
–4 4 – 8 – 4
0 4 1  0 0 1 17. AB =   [–1 2 1] = 
     3   – 3 6 3 
   

Applying R3  R3 – 4R2  – 1 4 – 3
 
2 –8 6 
 (AB) = 
 1 – 4 3 

39
 – 1 1 a 1 1
 
2 a b c
Now A = [1 – 4 3] and B =  
 1  1 1 b 1
= abc a b c
1 1 1 c
 – 1  – 1 4 – 3
    a b c
2  2 –8 6 
 B A =   [1 – 4 3] =
 1   1 – 4 3 
1 1 1 1 1
Hence (AB) = BA. 1
 
a b c b c
18. The order of matrix A = 3 × 4 1 1 1 1 1
1   1
The order of matrix B = 4 × 3 = abc a b c b c
 The order of matrix (AB) = 3 × 3 1 1 1 1 1
1   1
a b c b c
1 bc a(b  c )
(Operate : C1  C1 + C2 + C3)
1 ca b(c  a)
19. L.H.S. = 1 1
1 ab c(a  b) 1
b c
Applying C2  C2 + C3 1 1
 1 1 1  1 1
= abc  1     b c
1 bc  ab  ac ab  ac  a b c 1 1
1 1
1 ca  bc  ab bc  ab b c
 = = (ab + bc + ca)
1 ab  ca  bc ca  bc  1 1 1
Taking 1     common from C1
1 1 ab  ac  a b c
1 1 bc  ab
= (ab + bc + ca) × 0 = 0 1 1
1 1 ca  bc 1
b c
[ The elements of C1 and C2 are same] 0 1 0
= (abc + bc + ca + ab)
0 0 1
ax ax ax 3a  x a  x a  x

20. 0 =
ax ax ax
ax ax ax STUDYPIVOT.COM=
3a  x a  x a  x
3a  x a  x a  x
(Operate : R2  R2 – R1; R3  R3 – R1)
1 0
(C1  C1 + C2 + C3) = (abc + bc + ca + ab).1.
0 1
1 ax ax
= (abc + bc + ca + ab).
1 ax ax
= (3a – x)
1 ax ax
abc 2a 2a
1 ax ax 2b bc a 2b
22. Let  =
0 2x 0 2c 2c c ab
= (3a –x)
0 0 2x
Operate : R1  R1 + R2 + R3
(R2  R2 – R1; R3  R3 – R1)
abc abc abc
2x 0 2b bc a 2b
= (3a – x).1. (Expand by C1) =
0 2x 2c 2c c ab
= (3a – x) (4x2)  x = 0 , 3a.
1 1 1
1 a 1 1
2b b  c  a 2b
1 1 b 1 = (a + b + c)
21. L.H.S. = 2c 2c c ab
1 1 1 c
Operate : C2  C2 – C1 , C3  C3 – C1

1 0 0
2b  (b  c  a) 0
= (a + b + c)
2c 0  (c  a  b )
Expand by R1
40
25. Applying C3  C3 + (sin)C1 – (cos)C2, we have
 (a  b  c ) 0
= (a + b + c) sin  cos  0
0  (a  b  c )
= (a + b + c) [(a + b + c)2 – 0] = (a + b + c)3. sin  cos  0
=0
sin  cos  0
1 1 1 [ cos( + ) = cos cos  – sin sinetc.
a b c and C1 = 0]
23. L.H.S. =
a3 b3 c3
x4 2x 2x
Operate : C1 C1 – C2 ; C2  C2 – C3
2x x4 2x
26. LHS =
0 0 1 2x 2x x4
a–b b–c c
= Applying C1  C1 + C2 + C3
a3 – b3 b3 – c 3 c3
5 x  4 2x 2x
0 0 1 5x  4 x  4 2x
=
a–b b–c c 5x  4 2x x4
=
(a – b)(a 2  ab  b2 ) (b – c )(b2  bc  c 2 ) c 3
1 2x 2x
0 0 1
1 x4 2x
1 1 c = (5x + 4)
= (a – b) (b – c) 1 2x x4
a 2  ab  b 2 b 2  bc  c 2 c3
Applying R2  R2 – R1, R3  R3 – R1
1 1
= (a – b) (b – c) 1 2x 2x
a 2  ab  b 2 b 2  bc  c 2
0 4–x 0
[open w.r.t. R] = (5x + 4)
0 0 4–x
= (a – b) (b – c) [(b2 + bc + c2) – (a2 + ab + b2)]
= (a – b) (b – c) [(c2 – a2) + (bc – ab)] = (5x + 4) [1[(4 – x)2 – 0] – 0 + 0] = (5x + 4 (4 –
= (a – b) (b – c) (c – a) (a + b + c) = R.H.S. x)2

24. LHS.
STUDYPIVOT.COM
ab bc c a
bc c a ab 27.
x x2
y y 2
yz
zx
c a ab bc z z2 xy

operate : C1  C1 + C2 + C2 and take 2 common from Applying R2  R2 – R1, R3  R3 – R1


C1
x x2 yz
abc bc c a 2 2
y–x y –x zx – yz
abc c a ab 2 2
=2 z–x z –x xy – yz
abc ab bc
operate : C2  C2 – C1 ; C3  C3 – C1 x x2 yz
abc –a –b y – x ( y  x )( y – x ) z( x – y )
=
abc –b –c z – x ( z  x )( z – x ) – y( z – x )
=2
abc –c –a
operate : C1  C1 + C2 + C3 x x2 yz
– 1 – ( y  x) z
c –a –b c a b = (x – y) (z –x)
1 zx –y
a –b –c a b c
=2 = 2(–1) (–1)
b –c –a b c a Applying R3  R3 + R2

Interchanging C1 C2 and then C2  C3 x x2 yz


a b c –1 –y–x z
= (x – y) (z – x)
b c a 0 zx–y–x –yz
=2 = R.H.S
c a b

41
x x2 yz 1 0 – 2b
–1 –y–x z 0 1 2a
= (x – y) (z – x) = (1 + a2 + b2)2
0 – ( y – z ) – ( y – z) b – a 1 – a2 – b2

= (x – y) (z – x) (y – z) [x(y + x + z) + 1(– x2 + yz) Applying R3  R3 – bR1 + aR2


= (x – y) (y – z) (z – x) (xy + x2 + zx – x2 + yz) 1 0 – 2b
= (x – y) (y – z) (z – x) (xy + yz + zx)
0 1 2a
= (1 + a2 + b2)2
0 0 1  a2  b2
a2  1 ab ac
28. L.H.S. = ab b2  1 bc = (1 + a2 + b2)2 [1(1 + a2 + b2 – 0)]
ca cb 2
c 1 = (1 + a2 + b2)3 = R.H.S.

1 1 p 1 p  q
1 1 1
Applying R1  R1, R2  R2 and R3  R3 2 3  2p 1  3p  2q
a b c 30. L.H.S. =
3 6  3p 1  6p  3q
1 Applying R1  R2 – 2R1, R3  R3 – 3R1
a b c
a
1 1 1 p 1 p  q
=abc a b c
b 0 1 p –1
1 =
a b c 0 3 3p – 2
c
Expanding along C1, we get
Applying C1  aC1, C2  bC2 and C3  C3 = 1[(3p – 2) – (3p – 3)]
= 3p – 2 – 3p + 3 = 1 = R.H.S
a2  1 b2 c2 31. Minor of a23 is
abc
a2 b2  1 c2
= abc 2 2 2 2 –3
a b c 1 M23 = = 10 + 3 = 13
1 5
Applying C1  C1 + C2 + C3
32. We have |3A| = K|A|

STUDYPIVOT.COM
1  a2  b2  c 2 b2 c2 33 |A| = K|A| [ |KA| = Kn |A|]
1  a2  b2  c 2 b2  1 c2  K = 27
=
1  a2  b2  c 2 b2 c2  1 33.  |A| = 7 and order of matrix is 2
2 2  |adj A| = 72 = 49 [ |adj A| = |A|n–1]
1 b c
2
1 b 1 c2 a b c
= (1 + a2 + b2 + c2)
1 b2 c2  1 b c a
34. We have  =
Applying R2  R2 – R1 and R3  R3 – R1 c a b
1 b2 c2 Applying C1  C1 + C2 + C3, we get
1 1 0 abc b c
= (1 + a2 + b2 + c2)
0 0 1 bca c a
=
Expanding along C1, we get cab a b
= (1 + a2 + b2 + c2) [1(1 – 0)]
= 1 + a2 + b2 + c2 = R.H.S. 1 b c
1 c a
1  a2 – b2 2ab – 2b = (a + b + c)
1 a b
29. L.H.S. = 2ab 1 – a2  b2 2a
2b – 2a 1 – a2 – b2 Applying R2  R2 – R1, R3  R3 – R1, we get

Applying C1  C1 –bC3, C2  C2 + aC3 1 b c


0 c–b a–c
1  a2  b2 0 – 2b = (a + b + c)
0 a–b b–c
2 2
0 1 a  b 2a
= = (a + b + c) [(c – b) (b – c) – (a – b) (a – c)]
b(1  a 2  b 2 ) – a(1  a 2  b 2 ) 1 – a 2 – b 2
[Expanding along C1 ]
= (a + b+ c) [(bc – c2 – b2 + bc) – (a2 – ac – ab + bc)]
42
= (a + b + c) [2bc – c2 –b2 – a2 +ac + ab – bc]
= (a + b + c) [– a2 – b2 – c2 + ab + bc + ca]
= –(a + b + c) [a2 + b2 +c2 – ab – bc –ca]
= – (a3 + b3 + c3 – 3abc)
35. The given equations are
2 3 10 4 6 5 6 9 20
  = 4, –  = 1,  – =2
x y z x y z x y z
Given equation can be written as AX = B where

2 3 10  1/ x   4
     
4 –6 5  1/ y  and B =  1
A=  ,X=
6 9 – 20 1/ z  2
| A | = [2(120 – 45) – 3(– 80 – 30) + 10(36 + 36)]
= [150 + 330 + 720] = 1200
A11 = (120 – 45) = 75
A12 = – (80 – 30) = 110
A13 = + (36 + 36) = 72
A21 = – (– 60 – 90) = 150
A22 = + (– 40 – 60) = – 100
A23 = 0
A31 = + (15 + 60) = 75
A32 = – (10 – 40) = 30
A33 = + (– 12 – 12) = –24
1
A–1 = × adj. A
|A|

 75 150 75 
1  
=  110 – 100 30 
1200
 72 0 – 24
 X = A–1B. STUDYPIVOT.COM
 75 150 75   4
1    
=  110 – 100 30  ·  1
1200  72 0 – 24 2

1 1
= x=2
x 2
1 1
= y=3
y 3
1 1
= z=5
z 5

43
Chapter - 2
RELATIONS & BINARY OPERATIONS

RELATIONS

TYPES OF RELATIONS :

In this section we intend to define various types of relations on a given set A.


(i) Void relation : Let A be a set. Then   A × A and so it is a relation on A. This relation is called
the void or empty relation on A.

(ii) Universal relation : Let A be a set. Then A × A  A × A and so it is a relation on A. This relation
is called the universal relation on A.

(iii) Identity relation : Let A be a set. Then the relation IA = {(a, a) : a  A} on A is called the identity
relation on A. In other words, a relation IA on A is called the identity relation if every element of A
is related to itself only.

(iv) Reflexive relation : A relation R on a set A is said to be reflexive if every element of A is related
to itself. Thus, R on a set A is not reflexive if there exists an element a  A such that (a, a)  R.

(v)
STUDYPIVOT.COM
Note : Every identity relation is reflexive but every reflexive relation in not identity.
Symmetric relation : A relation R on a set A is said to be a symmetric relation
iff (a, b)  R  (b ,a)  R for all a, b  A. i.e. a R b  b R a for all a, b  A.

(vi) Transitive relation : Let A be any set. A relation R on A is said to be a transitive relation
iff (a, b)  R and (b, c)  R  (a, c)  R for all a, b, c  A i.e. a R b and b R c  a R c
for all a, b, c  A

(vii) Equivalence relation : A relation R on a set A is said to be an equivalence relation on A iff


(i) it is reflexive i.e. (a, a)  R for all a  A
(ii) it is symmetric i.e. (a, b)  R  (b, a)  R for all a, b  A
(iii) it is transitive i.e. (a, b)  R and (b, c)  R  (a, c)  R for all a, b  A

Example # 1 : Which of the following are identity relations on set A = {1, 2, 3}.
R1 = {(1, 1), (2, 2)}, R2 = {(1, 1), (2, 2), (3, 3), (1, 3)}, R3 = {(1, 1), (2, 2), (3, 3)}.
Solution: The relation R3 is idenity relation on set A.
R1 is not identity relation on set A as (3, 3)  R1.
R2 is not identity relation on set A as (1, 3)  R2

Example # 2 : Which of the following are reflexive relations on set A = {1, 2, 3}.
R1 = {(1, 1), (2, 2), (3, 3), (1, 3), (2, 1)}, R2 = {(1, 1), (3, 3), (2, 1), (3, 2)}..
Solution : R1 is a reflexive relation on set A.
R2 is not a reflexive relation on A because 2  A but (2, 2)  R2.

Example # 3 : Prove that on the set N of natural numbers, the relation R defined by x R y  x is less than y is
transitive.
Solution : Because for any x, y, z  N x < y and y < z  x < z  x R y and y R z  x R z. so R is
44
transitive.

Example # 4 : Let T be the set of all triangles in a plane with R a relation in T given by
R = {(T1 , T2) : T1 is congruent to T 2}. Show that R is an equivalence relation.
Solution : Since a relation R in T is said to be an equivalenece relation if R is reflexive, symmetric and
transitive.
(i) Since every triangle is congruent to itself
 R is reflexive
(ii) (T1 , T2)  R  T1 is congruent to T2
 T2 is congruent to T1  (T2, T1)  R
Hence R is symmetric
(iii) Let (T1, T2)  R and (T2, T3)  R  T1 is congruent to T2
and T2 is congruent to T3
 T1 is congruent to T3
 (T1, T3)  R
 R is transitive
Hence R is an equivalence relation.

Example # 5 : Show that the relation R in R defined as R = {(a, b) : a  b} is transitive.


Solution : Let (a, b)  R and (b, c)  R
 (a  b) and b  c  a  c
 (a, c)  R
Hence R is transitive.

Example # 6 : Show that the relation R in the set {1, 2, 3} given by R = {(1, 2), (2, 1)} is symmetric.
Solution : Let (a, b)  R [ (1, 2)  R]
 (b, a)  R [ (2, 1)  R]
Hence R is symmetric.

(1)
STUDYPIVOT.COM
Self Practice Problem :
Let L be the set of all lines in a plane and let R be a relation defined on L by the rule (x ,y)  R
 x is perpendicular to y. Then prove that R is a symmetric relation on L.

(2) Let R be a relation on the set of all lines in a plane defined by (1, 2)  R  line 1 is parallel to
line 2. Prove that R is an equivalence relation.
BINARY OPERATION
BINARY OPERATION
A binary operation * on a set A is a function * : A × A  A. We denote * (a, b) by a * b.
Example # 1 : Show that addition, subtraction and multiplication are binary operations on R, but division is not a
binary operation on R. Further, show that division is a binary operation on the set R* of non zero real
numbers.
Solution : + : R × R  R is given by
(a, b)  a + b
– : R × R  R is given by
(a, b)  a – b
× : R × R  R is given by
(a, b)  ab
Since ‘+’, ‘–’ and ‘×’ are functions, they are binary operations on R.
a
But  : R × R  R, given by (a, b)  , is not a function and hence not a binary operation, as for
b
a
b = 0, is not defined.
b

45
a
However  : R* × R*  R*, given by (a, b)  is a function and hence a binary operation on R *.
b
Example # 2 : Show that subtraction and division are not binary operations on N.
Solution : – : N × N  N, given by (a, b)  a – b is not binary operation as the image of (3, 5) under ‘–’ is
3 – 5 = –2  N. Similarly  : N × N  N given by (a, b)  a  b is not a binary operation as the image
3
of (3, 5) under  is 3  5 =  N .
5

Example # 3 : Show that * : R × R  R given by (a, b)  a + 4b2 is a binary operation.


Solution : Since * carries each pair (a, b) to a unique element a + 4b2 in R, * is a binary operation on R.

Example # 4 : Show that the  : R × R  given by (a, b)  max {a, b} and  : R × R  R given by (a, b)  min
{a, b} are binary operations.
Solution : 
Since carries each pair (a, b) in R × R to a unique element namely maximum of a and b lying in
R,  is a binary operation. Using the similar argument, one can say that  is also a binary
operation.

OPERATION TABLE

When number of elements in a set A is small, we can express a binary operation * on the set A through a
table called the operation table for the operation *. For example consider A = {1, 2, 3}. Then the operation *
on A defined as * : R × R  given by (a, b)  max {a, b} can be expressed by the following operation table.
Here *(1, 3) = 3, *(2, 3) = 3, *(1, 2) = 2.
* 1 2 3
1 1 2 3

STUDYPIVOT.COM 2 2 2 3
3 3 3 3

LAWS OF BINARY OPERATIONS :

(i) Commutative Law : A binary operation * on the set X is called commutative


if a * b = b * a for every a, b  X.
(ii) Associative Law : A binary operation * : A × A  A is said to be associative
if (a * b) * c = a * (b * c)  a, b, c  A.

Example # 5 : : Show that + : R × R  R and × : R × R  R are commutative binary operations but – : R × R  R


and  : R × R  R are not commutative.
Solution : Since a + b = b + a and a × b = b × a  a b  R so '+' and '×' are commutative binary operations.
However '–' is not commutative since 3 – 4  4 – 3.
Similarly 3  4  4  3 shows that  is not commutative.

Example # 6 : Show that * : R × R  R defined by a * b = a + 2b is not commutative.


Solution : Since 3 * 4 = 3 + 8 = 11 and 4 * 3 = 4 + 6 = 10, showing that the operation * is not commutative.

Example # 7 : Show that addition and multiplication are associative binary operations on R. But subtraction is not
associative on R. Division is not associative on R.
Solution : Addition and multiplication are associative since (a + b) + c = a + (b + c) and (a × b) × c
= a × (b × c) 
a, b, c  R. However subtraction and division are not associative as (8 – 5) – 3  8 – (5 – 3) and
(8  5) 3  8  (5  3).

Example # 8 : Show that * : R × R  R given by a * b  a + 2b is not associative.


Solution : The operation * is not associative since
(8 * 5) * 3 = (8 + 10) * 3 = (8 + 10) + 6 = 24.

46
while 8 * (5 * 3) = 8 * (5 + 6) = 8 * 11 = 8 + 22 = 30.

EXISTENCE OF IDENTITY ELEMENT :

Given a binary operation * : A × A  A an element e  A if it exists is called identity for the operation *
if a * e = a = e * a  a  A.

Example # 9 : Show that zero is the identity for addition on R but 1 is the identity for multiplicaton on R. But there
is no identity element for the operations – : R × R  R and  : R* × R*  R*.
Solution. a + 0 = 0 + a = a and a × 1 = a = 1 × a  a  R implies that 0 and 1 are identity elements for the
operations '+' and '×' respectively. Further there is no element e in R with a – e = e – a  a. Similarly
we can not find any element e in R* such tht a  e = e  a  a in R*. Hence, '–' and '' do not have
identity element.

EXISTENCE OF INVERSE :

Given a binary operation * : A × A  A with the identity element e in A, an element a  A is said to be invertible
with respect to the operation * if there exists an element b in A such that a * b = e = b * a and b is called the
inverse of a and is denoted by a–1.

Remark :Zero is identity for the addition operation on R but it is not identity for the addition operation on N as 0  N.
In fact the addition operation on N does not have any identity.
One further notices that for the addition operation + : R × R  R, given any a  R, there exists – a in R such
that a + (–a) = 0 (identity for '+') = (–a) + a.
Similarly for the multiplication operation on R given any a  0 in R, we can choose 1/a in R such that
a × 1/a = 1(identity for '×') = 1/a × a. This leads to the existence of inverse.

Solution :
STUDYPIVOT.COM
Example # 10 : Show that –a is the inverse of a for the addition operation '+' on R and 1/a is the inverse of a  0 for
the multiplication operation '×' on R.
As a + (–a) = a – a = 0 and (–a) + a = 0, –a is the inverse of a for addition.
1 1 1
Similarly for a  0 a × =1= × a implies that is the inverse of a for multiplication.
a a a
1
Example # 11 : Show that –a is not the inverse of a  N for the addition operation + on N and is not the inverse
a
of a  N for multiplication operation × on N for a  1.
Solution : Since –a  N, –a can not be inverse of a for addition operation on N. Althrough –a satisfies
a + (–a) = 0 = (–a) + a.
1
Similarly for a  1 in N,  N which implies that other than 1, no element of N has inverse for
a
multiplication operation on N.

47
EXERCISE - 1: IMPORTANT BOARD LEVEL QUESTIONS
Type (I) : Very Short Answer Type Questions : [01 Mark Each]

1. Show that the relation R in R defined as R = {(a, b) : a  b} is not symmetric.

2. Show that the relation R in the set {1, 2, 3}, given by R = {(1, 2), (2, 1)} is not reflexive.

3. Show that the binary operation * defined by a * b = ab + 1 on Q is commutative.

4. If the binary operation * on the set of integers Z, is defined by a * b = a + 3b 2, then find the value 2 * 4.

Type (II) : Short Answer Type Questions : [02 Marks Each]

5. Show that the binary operation * defined by a * b = a – b on z is not commutative and associative.

6. Is * defined on the set {1, 2, 3, 4, 5} by a * b = l.c.m. of a and b binary operation ?

Type (III) : Long Answer Type Questions : [04 Mark Each]

7. Check whether the relation R defined in the set {1, 2, 3, 4, 5, 6} as R = {(a, b) : b = a + 1} is reflexive,
symmetric or transitive.

8. STUDYPIVOT.COM
Show that the binary operation * defined by a * b = ab + 1 on Q is not associative.

9. Let * be the binary operation on N given by a * b = LCM of a and b. Find the value of 20 * 16. Is * (i)
commutative (ii) associative ?

Type (IV) : Very Long Answer Type Questions: [06 Mark Each]

10. Show that the relation R in the set (1, 2, 3) given by R = {(1, 1), (2, 2), (3, 3), (1, 2), (2, 3)} is reflexive but
neither symmetric nor transitive.

11. Show that addition, subtraction and multiplication are binary operations on R, but division is not binary
operation on R.

ab
12. (i) Is the binary operation * defined on set N, given by a*b = for all a, b  Q, commutative ?
2
(ii) Is the above binary operation * associative ?

3ab
13. Let * be a binary operation on Q defined by a * b = , Show that * is commutative as well as associative.
5

Also find its identity element, if it exists.

48
EXERCISE - 2: CBSE PREVIOUS YEARS QUESTIONS

1. Show that the relation R defined by (a, b) R(c, d)  a + d = b + c on the set N × N is an equivalence relation.
[CBSE 2008]

2. Let * be a binary operation on the set Q of rational numbers given as a * b = (2a – b) 2, a, b  Q. Find 3 * 5 and
5 * 3. Is 3 * 5 = 5 * 3 ? [CBSE 2008]

3. Prove that the relation R in the set A = {1, 2, 3, 4, 5} given by R = {(a, b) : |a – b| is even}, is an equivalence
relation. [CBSE 2009]

4. Let * be a binary operation on N given by a * b = HCF (a, b), a, b  N. Write the value of 22 * 4.
[CBSE 2009]
ab
5. Let * be a binary operation on set Q of rational numbers defined as a * b = . Write the identity for *, if any
5
[CBSE 2009]

6. Let Z be the set of all integers and R be the relation on Z defined as R = {(a, b) : a, b  Z, and (a – b) is divisible
by 5} Prove that R is an equivalence relation. [CBSE 2010]

7. Show that the relation S in the set R of real numbers, defined as S = {(a, b) : a, b  R and a  b3} is neither
reflexive, nor symmetric nor transitive. [CBSE 2010]

8. Show that the relation S in the set A = {x  Z : 0  x  12} given by S = {(a, b) : a, b  Z, |a – b| is divisible by

STUDYPIVOT.COM
4} is an equivalence relation. Find the set of all elements related to 1. [CBSE 2010]
9. Show that the relation S defined on the set N × N by (a, b) S(c, d)  a + d = b + c is an equivalence relation.
[CBSE 2010]

10. Let f : X  Y be a function. Define a relation R on X given by R = {(a, b) : f(a) = f(b)}. Show that R is an
equivalence relation on X. [CBSE 2010]

11. State the reason for the relation R in the set {1, 2, 3} given by R = {(1, 2), (2, 1)} not to be transitive.
[CBSE 2011]
12. If * be a binary operation on set of integers I defined by [CBSE 2011]
a * b = 3a + 4b – 2, find the value of 4 * 5.

13. Let * be a binary operation on set of integers , defined by [CBSE 2011, 2008]
a * b = 2a + b – 3. Find the value of 3 * 4

14. Consider the binary operation * on the set {1, 2, 3, 4, 5} defined by a * b = min {a, b}. Write the operation table
of the operation *. [CBSE 2011]
 a  b, if ab  6
15. A binary operation * on the set {0, 1, 2, 3, 4, 5} is defined as : a * b = 
a  b – 6, if ab  6
Show that zero is the identity for this operation and each element ‘a  0’ of the set is invertible with 6 – a,
being the inverse of ‘a’. [CBSE 2011]

16. Consider the binary operations * : R × R  R and o : * : R × R  R defined as a * b = |a – b| and a o b = a


for all a, b R. Show that '*' is commutative but not associative 'o' is associative but not commutative.
[CBSE 2012]
17. If the binary operation * on the set Z of integers is defined by a * b = a + b – 5, then write the identity element
for the operation * in Z. [CBSE 2012]

18. The binary operation * : R × R  R, is defined as a * b = 2a + b. Find (2 * 3) * 4. [CBSE 2012]

49
SOLUTIONS
 (a * b) * c = LCM(a, b) * c = LCM(a, b, c)
EXERCISE - 1 and a * (b * c) = a * LCM (b, c) = LCM (a, b, c)
 (a * b) * c = a * (b * c)
1. Let (a, b)  R Hence * is associative.
 ab   ba
 (b, a)  R 10. Since (1, 1), (2, 2) and (3, 3) lie in R
Hence R is not symmetric.  R is reflexive.
Also (1, 2)  R but (2, 1)  R
2. Since (1, 1)  R also (2, 2)  R and (3, 3)  R  R is not symmetric
since (1, 2)R and (2, 3)R but (1, 3)R
 (a, a)  R
 R is not transitive.
Hence R is not reflexive.
11. (+) : R × R  R is given by
3. Let a, b  Q
(a, b)  a + b
 a * b = ab + 1 = ab + 1 = b * a
(–) : R × R  R is given by
 a*b=b*a
(a, b)  a – b
Hence binary operation * is commutative.
and (×) : R × R  R is given by
4. Here a * b = a + 3b2 (a, b)  ab
 2 * 4 = 2 + 3 (4)2 = 2 + 48 = 50 Since ‘+’, ‘–’ and ‘×’ are functions
 They are binary operations on R
5. (i) Let a, b  z But () : R × R  R, given by
 a – b  b – a
a
 a*bb*a (a, b)  is not a function
Hence binary operation * is not commutative. b
(ii) Let a, b, c z a
since for b = 0, is not defined.
 (a * b) * c = ((a – b) * c b
=a–b–c  ‘’ is not a binary operation on R.

STUDYPIVOT.COM
and a * (b * c) = a * (b – c)
= a – (b – c) ab
12. (i)  a * b =
=a–b+c 2
 (a * b) * c  a * (b * c) ba
Hence binary operation * is not associative. b*a= [ a, b  Q]
2
6. a * b = l.c.m. of a and b a*b=b*a
 2 * 3 = l.c.m. of 2 and 3 Hence, ‘ * ’ is commutative
=6 [ l.c.m. of 2 and 3 = 2 × 3 = 6] (ii) Let a, b, c  Q
By 6{1, 2, 3, 4, 5} bc
Hence ‘*’ is not a binary operation on {1, 2, 3, 4, 5} bc  a
a * (b * c) =a*   = 2
7. Here R = {(a, b) : b = a + 1}  2  2
= {(a, a + 1) : a, a + 1  {1, 2, 3, 4, 5, 6}} 2a  b  c 2a  b  c
= {(1, 2) (2, 3), (3, 4), (4, 5), (5, 6)} = =
(i) R is not reflexive as (a, a)  R  a. 2. 2 4
(ii) R is not symmetric as (a, b)R but (b, a)R ab
(iii) R is not transitive as (a, b)R and (a, c)R but ab c
(a, c)R and (a * b) * c =   *c = 2
 (1, 2)R, (2, 3)R but (1, 3)R  2  2
a  b  2c
8. Let a, b, c  Q =
 a * (b * c) = a * (bc + 1) = a(bc + 1) + 1 4
= abc + a + 1 and (a * b) * c  (a * b) * c
Now (a * b) * c = (ab + 1) * c Hence ‘*’ is not associative.
= (ab + 1) c + 1 = abc + c + 1 13. (i) For all a, b  Q
 a * (b * c)  (a * b) * c
Hence the binary operation * is not associative. 3ab 3ba
a*b= = =b*a
5 5
9.  a * b = LCM of a and b = LCM (a, b)
 20 * 16 = LCM (20, 16) = 80  ‘*’ is commutative
(ii) For all a, b, c  Q
(i) Let a, b  N
 a * b = LCM (a, b) = LCM (b, a) = b * a  3ab  3  3ab  9
(a * b) * c =   *c=  c  abc
Hence * is commutative.  5  5  5  25
(ii) Let a, b, c  N

50
 3bc  3  3bc  9 ae
and a * (b * c) = a *   = a  abc  a*e=a  = a e = 5
 5  5  5  25 5
 (a * b) * c = a * (b * c) 6. Reflexive :  a – a is divisible by 5 for all a  Z
Hence ‘*’ is associative  R is reflexive
(iii) Let e  Q be an identity element Symmetric : (a, b)  R  a – b is divisible by 5
 such that e * a = a * e = a  b – a is divisible by 5
3 3  b–aR
e*a= ea = a and a * e = ae = a  R is symmetric
5 5 Transitive : (a, b)  R and (b, c)  R
3 3  a – b and b – c are both divisible by 5
 ea = a and e=1  a – b + b – c is divisible by 5
5 5
 a – c is divisible by 5
5  (a, c)  R
e= Q
3  R is transitive
5 Since R is reflexive, symmetric and transitive.
Hence identity element is Q Hence, R is an equivalence relation.
3
7. (i) As a  a3 is not true for all a  R
 R is not reflexive
EXERCISE - 2 1
For example, if a = then a > a3
3
CBSE-PREVIOUS YEARS i.e. a  a3 is not true
(ii) If (a, b)  R then need not imply that (b, a)  R
1. Reflexive  R is not symmetric
 a + b = b + a  (a, b) R(a, b) For example, if (1, 2)  R but (2, 1)  R,
Hence R is reflexive.
(ii) Symmetric As 1  23 but 2 13
(a, b) R(c, d)  a + d = b + c (iii) If (a, b)  R and (b, c)  R, then need not imply
b+c=a+d  c+b=d+a that (a, c)  R
 (c, d) R(a, b)  R is not transitive
Hence R is symmetric For example (100, 5)  R and (5, 2)  R but (100, 2)

STUDYPIVOT.COM
(iii) (a, b) R(c, d) and (c, d) R(e, f) R
 a + d = b + c and c + f = d + e As 100  53 and 5  23 but 100  23.
a+d+c+f=b+c+d+e 8. Reflexive : For all a  A
 a + f = b + e  (a, b) R(e, f) |a – a| = 0 is divisible by 4  (a, a)  S
Hence R is transitive  S is reflexive
Therefore, relation R is an equivalence relation. Symmetric : Let a, b  A, (a , b)  S
 |a – b| is divisible by 4
2. Here a * b = (2a – b)2
 |b – a| is divisibe by 4  (b, a)  S
3 * 5 = (6 – 5)2 = (1)2 = 1
5 * 3 = (10 – 3)2 = (7)2 = 49  S is symmetric
 3*55*3 Transitive : Let a, b, c  A, (a, b)  S, (b, c)  S
 |a – b| is divisible by 4 and |b – c| is divisible by 4
3. Reflexive : Let a  A
 (a – b) and (b – c) is divisible by 4
 |a – a| = 0 is an even number
 (a – b) + (b – c) = a – c is divisible by 4
 (a, a)  R
 R is reflexive  |a – c| is divisible by 4 (a, c) S
Symmetric : Let a, b  R  S is transitive
Let (a, b)  R Since S is reflexive, symmetric and transitive.
 |a – b| is even  |–(b – a)| is even Hence, S is an equivalence relation.
 |b – a| is even The set of all elements of A, related to 1 is {1, 5, 9}
 (b, a)  R
9. Reflexive
 R is symmetric
 a + b = b + a  (a, b) R(a, b)
Transitive : Let a, b, c  R
Hence R is reflexive.
Let (a, b)  R and (b , c)  R
(ii) Symmetric
 |a – b| is even and |b – c| is even
(a, b) R(c, d)  a + d = b + c
 a – b is even and b – c is even
b+c=a+d  c+b=d+a
 (a – b) + (b – c) is even
 (c, d) R(a, b)
 |a – c| is even  (a, c)  R
Hence R is symmetric
 R is transitive.
(iii) (a, b) R(c, d) and (c, d) R(e, f)
Hence R is an equivalence relation as R is reflex-
 a + d = b + c and c + f = d + e
ive, symmetric and transitive.
a+d+c+f=b+c+d+e
4. a * b = HCF (a, b)  a + f = b + e  (a, b) R(e, f)
 22 * 4 = HCF (22, 4) = 2 Hence R is transitive
5 . Let e is identity element for '*' Therefore, relation R is an equivalence relation.
51
10. Reflexive : For every a  X, since f(a) = f(a) commutative.
 (a, a)  R  R is reflexive Let a = 2, b = 3, c = 4
Symmetric : Let (a, b)  R for every a, b  X (a * b) * c = (2 * 3) * 4 = |2 – 3| * 4 = 1 * 4
 f(a) = f(b)  f(b) = f(a)  (b, a)  R = |1 – 4| = 3
 R is symmetric a * (b * c) = 2 * (3 * 4) = 2 * |3 – 4| = 2 * 1 = |2 – 1|
Transitive : Let a, b, c  X. Let (a, b)  R and (b, c) =1
 R  f(a) = f(b) and f(b) = f(c) As (a * b) * c  a * (b * c)
 f(a) = f(c)  (a, c)  R Hence * is not associative
 R is transitive. Consider binary operation a o b = a, a, b  R
Since R is reflexive, symmetric and transitive. a o b = a and b o a = b
Therefore R is an equivalence relation. As a o b  b o a. Hence o is not commutative.
11. In the case of transitive relation Also consider (a o b) o c = a o c = a and
(a, b), (b, c)  R  (a, c)  R a o (b o c) = a o b = a
Here (1, 2) and (2, 1)  R but (1, 1)  R. As (a o b) o c = a o (b o c) for all a, b, c  R
So, it is not transitive. Hence o is associative.
12. Here a * b = 3a + 4b – 2 (a, b  I) 17. Let e be identity element, then a * e = e * a = a
 4 * 5 = 3 × 4 + 4 × 5 – 2 = 12 + 20 – 2 = 30  a+e–5=e+a–5=a  e=5
13. Given : a * b = 2a + b – 3 (a, b  ) 18. Given a * b = 2a + b
 3 * 4 = 2 × 3 + 4 – 3 = 7. (2 * 3) * 4 = (4 + 3) * 4 = 7 * 4 = 14 + 4 = 18

* 1 2 3 4 5
1 1 1 1 1 1
14. 2 1 2 2 2 2
3 1 2 3 3 3
4 1 2 3 4 4
5 1 2 3 4 5

15. We construct the composition table as follows :

*
STUDYPIVOT.COM
0 1 2 3 4 5
0 0 1 2 3 4 5
1 1 2 3 4 5 0
2 2 3 4 5 0 1
3 3 4 5 0 1 2
4 4 5 0 1 2 3
5 5 0 1 2 3 4
From this table, it is clear that
0 * 0 = 0, 1 * 0 = 0 * 1 = 1, 2 * 0 = 0 * 2 = 2, 3 * 0
= 0 * 3 = 3, 4 * 0 = 0 * 4 = 4
and 0 * 5 = 5 * 0 = 5
Hence 0 is the identity element w.r.t. *
Also for each a  0 in {0, 1, 2, 3, 4, 5}
6 – a {0, 1, 2, 3, 4, 5}
and a * (6 – a) = a + (6 – a) – 6 = 0
Hence 6 – a is inverse of a for each a  0 in the set
{0, 1, 2, 3, 4, 5}. Also 0 * 0 = 0, therefore, 0 is inverse
of itselt. Now that 6 – a{0, 1, 2, 3, 4, 5} when a = 0.
Statement of the equation is not totally correct.
16. Consider binary operation a * b = |a – b|
a * b = |a – b|
and b * a = |b – a| = | – (a – b) | = |a – b|
As a * b = b * a for all a, b  R. Hence * is

52
Chapter - 3
FUNCTIONS
Definition :
Function is a rule (or correspondence), from a non empty set A to a non empty set B, that associates
each member of A to a unique member of B. Symbolically, we write f: A  B. We read it as "f is a
function from A to B".
For example, let A  {–1, 0, 1} and B  {0, 1, 2}.
Then A × B  {(–1, 0), (–1, 1), (–1, 2), (0, 0), (0, 1), (0, 2), (1, 0), (1, 1), (1, 2)}
Now, " f : A  B defined by f(x) = x 2 " is the function such that
f  {(–1, 1), (0, 0), (1, 1)}
f can also be shown diagramatically by following mapping.
A B

Note : Every function say y = f(x) : A  B. Here x is independent variable which takes its values from A
while 'y' takes its value from B. A relation will be a function if and only if
(i) x must be able to take each and every value of A and
and (ii) one value of x must be related to one and only one value of y in set B.

STUDYPIVOT.COM
Graphically : If any vertical line cuts the graph at more than one point, then the graph does not
represent a function.
Example # 1 : (i) Which of the following correspondences can be called a function ?
(A) f(x) = x3 ; {–1, 0, 1}  {0, 1, 2, 3}
(B) f(x) = ± x ; {0, 1, 4}  {–2, –1, 0, 1, 2}
(C) f(x) = x ; {0, 1, 4}  {–2, –1, 0, 1, 2}
(D) f(x) = – x ; {0, 1, 4}  {–2, –1, 0, 1, 2}
(ii) Which of the following pictorial diagrams represent the function

(A) (B)

(C) (D)

Solution :
(i) f(x) in (C) and (D) are functions as definition of function is satisfied. while in case of (A) the
given relation is not a function, as f(–1)  2nd set. Hence definition of function is not satisfied.
While in case of (B), the given relation is not a function, as f(1) = ± 1 and f(4) = ± 2 i.e. element
1 as well as 4 in 1st set is related with two elements of 2nd set.Hence definition of function is not
satisfied.

54
(ii) B and D. In (A) one element of domain has no image, while in (C) one element of 1st set has
two images in 2nd set

Self practice problem :


(1) Let g(x) be a function defined on [1, 1]. If the area of the equilateral triangle with two of its
vertices at (0,0) and (x,g(x)) is 3 / 4 sq. unit, then the function g(x) may be.
2
(A) g(x)=  (1  x ) (B) g(x) = (1  x 2 ) (C) g(x) =  (1  x 2 ) (D) g(x) = (1  x 2 )

(2) Represent all possible functions defined from {} to {1, 2}.
Answers : (1) B, C

(2) (i) (ii) (iii) (iv)

Domain, Co-domain and Range of a Function :

Let y = f(x) : A  B, then the set A is known as the domain of f and the set B is known as co-domain
of f.

STUDYPIVOT.COM
If x 1 is mapped to y1, then y1 is called as image of x 1 under f. Further x 1 is a pre-image of y1 under f.
If only expression of f (x) is given (domain and co-domain are not mentioned), then domain is complete
set of those values of x for which f (x) is real, while codomain is considered to be (– , )
(except in inverse trigonometric functions).
Range is the complete set of values that y takes. Clearly range is a subset of Co-domain.
A function whose domain and range are both subsets of real numbers is called a real function.

Example # 2 : Find the domain of following functions :


(i) f(x) = x2  5 (ii) sin–1 (2x – 1)

x 2  5 is real iff x – 5  0
Solution : (i) f(x) = 2

 |x|  5  x  – 5 or x  5

 the domain of f is (–, – 5 ]  [ 5 , )

(ii) sin–1 (2x – 1) is real iff –1  2x – 1  + 1


 domain is x  [0, 1]

Algebraic Operations on Functions :


If f and g are real valued functions of x with domain set A and B respectively, then both f and g are
defined in A  B. Now we define f + g, f  g, (f . g) and (f /g) as follows:

f f( x )
  (x) =
(iii)
g g( x ) domain is {x  x  A  B such that g(x)  0}.

55
Note :  For domain of (x) = {f(x)}g(x) , conventionally, the conditions are f(x) > 0 and g(x) must be real.
 For domain of (x) = f(x)Cg(x) or (x) = f(x)Pg(x) conventional conditions of domain are f(x)  g(x)
and f(x)  N and g(x)  W

Example # 3 : Find the domain of following functions :

(i) f(x) = sin x  16  x 2


3
(ii) f(x) = log(x3  x)
4  x2
1
(iii) f(x) = x cos x

Solution : (i) sin x is real iff sin x  0  x[2n, 2n + ], nI.

16  x 2 is real iff 16  x  0   4  x  4.
2

Thus the domain of the given function is {x : x[2n, 2n + ], nI }[4, 4]=[4, ]  [0, ].

(ii) Domain of 4  x 2 is [2, 2] but 4  x 2 = 0 for x = ± 2  x  (–2, 2)


log(x 3  x) is defined for x3  x > 0 i.e. x(x  1)(x + 1) > 0.
 domain of log(x 3  x) is (1, 0 )  (1, ).
Hence the domain of the given function is {(1, 0 )  (1, )} (2, 2)  (1, 0 )  (1, 2).

(iii) x > 0 and –1  x  1


 domain is (0, 1]

Self practice problems :

(3)

(i)
STUDYPIVOT.COM
Find the domain of following functions.
1
f(x) = log( 2  x ) + x  1 (ii) f(x) = 1 x – sin
–1 2x  1
3

Answers : (i) [–1, 1)  (1, 2) (ii) [–1, 1]

Methods of determining range :

(i) Representing x in terms of y


If y = f(x), try to express asx = g(y), then domain of g(y) represents possible values of y,
which is range of f(x).

x2  x  1
Example # 4 : Find the range of f(x) =
x2  x  1
2
x  x 1
Solution : f(x) = {x 2 + x + 1 and x2 + x – 1 have no common factor}
x2  x  1
x2  x  1
y=
x2  x  1
 yx 2 + yx – y = x2 + x + 1
 (y – 1) x 2 + (y – 1) x – y – 1 = 0
If y = 1, then the above equation reduces to –2 = 0. Which is not true.
Further if y  1, then (y – 1) x 2 + (y – 1) x – y – 1 = 0 is a quadratic and has real roots if
(y – 1)2 – 4 (y – 1) (–y – 1)  0
i.e. if y  –3/5 or y  1 but y  1
Thus the range is (–, –3/5]  (1, )

56
(ii) Graphical Method :
The set of y– coordinates of the graph of a function is the range.

x2  4
Example # 5 : Find the range of f(x) =
x2

x2  4
Solution : f(x) = = x + 2; x  2
x2

 graph of f(x) would be


Thus the range of f(x) is R – {4}
Further if f(x) happens to be continuous in its domain
then range of f(x) is [min f(x), max. f(x)]. However for
sectionally continuous functions, range will be union
of [min f(x), max. f(x)] over all those intervals where
f(x) is continuous, as shown by following example.

Example # 6 : Let graph of function y = f(x) is

STUDYPIVOT.COM
Then range of above sectionally continuous function is [y2, y3]  (y4, y5]  (y7, y6]

(iii) Using monotonocity : Many of the functions are monotonic increasing or monotonic decreasing. In
case of monotonic continuous functions the minimum and maximum values lie at end points of domain.
Some of the common function which are increasing or decreasing in the interval where they are
continuous is as under.

For monotonic increasing functions in [a, b]


(i) f(x)  0
(ii) range is [f(a), f(b)]
for monotonic decreasing functions in [a, b]
(i) f(x)  0
(ii) range is [f(b), f(a)]

57
Example # 7 : Find the range of following functions :

(i) y = n (2x – x 2) (ii) y = sec –1 (x 2 + 3x + 1)

Solution : (i) Step – 1


We have 2x – x 2  (–, 1]
Step – 2 Let t = 2x – x 2
For nt to be defined accepted values are (0, 1]
Now, using monotonocity of n t,
n (2x – x 2)  (–, 0]
 range is (– , 0] Ans.
(ii) –1 2
y = sec (x + 3x + 1)

 5 
Let t = x 2 + 3x + 1 for x  R, then t   ,  
 4 

 5 
but y = sec–1 (t)  t   ,  1  [1, )
 4 

STUDYPIVOT.COM
   1 5  
from graph the range is 0,   sec   4 , 
 2    

Self practice problems :

(4) Find domain and range of following functions.

x 2  2x  5 1
(i) y = x3 (ii) y= 2 (iii) y=
x  2x  5 x2  x

 1 2 3 
(iv) y = cot–1 (2x – x2) (v) y = n  sin  x  x   
  4 

3  5 3  5 
Answers : (i) domain R; range R (ii) domain R ; range  , 
 2 2 

 
(iii) domain R – [0, 1] ; range (0, ) (iv) domain R ; range  ,  
4 

 2  8  2  8    
(v) domain x   ,  ; range n , n 
 4 4   6 2

Various Types of Functions :


(i) Polynomial Function :

If a function f is defined by f (x) = a0 x n + a1 x n1 + a2 x n2 +... + an1 x + an, where n is a non
negative integer and a0, a1, a2,........., an are real numbers and a0  0, then f is called a
polynomial function of degree n.
58
Note :  There are only two polynomial functions, satisfying the relation; f(x).f(1/x) = f(x) + f(1/x), which
are f(x) = 1  x n

 1 a0 a1
Proof : Let f(x) = a0 x n + a1 xn – 1 + ......... + an, then f   = n + n  1 + ......... + an.
x x x

Since the relation holds for many values of x,


 Comparing the coefficients of xn, we get a0 an = a0  an = 1
Similary comparing the coefficients of x n – 1, we get a0 an – 1 + a1 an = a1  an – 1 = 0,
like wise an – 2 , ......, a1 are all zero.

Comparing the constant terms, we get a 02  a12  .......  a n2 = 2 a n2  a0 = ±1

(ii) Algebraic Function :

y is an algebraic function of x, if it is a function that satisfies an algebraic equation of the


form, P0 (x) yn + P1 (x) yn1 +....... + Pn1 (x) y + Pn (x) = 0, where n is a positive integer and
P0 (x), P1 (x)....... are polynomials in x. e.g. y = x is an algebraic function, since it satisfies
the equation y²  x² = 0.
Note :  All polynomial functions are algebraic but not the converse.

 A function that is not algebraic is called Transcendental Function.


(iii) Rational Function :
g( x )
A rational function is a function of the form, y = f (x) = , where g (x) and h (x), h(x)  0
h( x )
are polynomials.

(iv)
STUDYPIVOT.COM
Exponential Function :

A function f(x) = ax = ex In a (a > 0, a  1, x  R) is called an exponential function. Graph of


exponential function can be as follows :
Case -  Case - 
For a > 1 For 0 < a < 1

(v) Logarithmic Function : f(x) = logax is called logarithmic function, where a > 0 and a  1 and
x > 0. Its graph can be as follows
Case-  Case- 
For a > 1 For 0 < a < 1

59
(vi) Absolute Value Function / Modulus Function :
 x if x0
The symbol of modulus function is f (x) = x and is defined as: y = x  
 x if x0

(vii) Signum Function : (Also known as sgn(x))


A function f (x) = sgn (x) is defined as follows :

 1 for x  0

f (x) = sgn (x) =  0 for x  0
 1 for x  0

| x |
 ; x0
It is also written as sgn x =  x
 0 ; x  0

| f ( x) |
 ; f ( x)  0
Note : sgn f(x) =  f ( x )
 0 ; f (x)  0

(viii) Greatest Integer Function or Step Function :

STUDYPIVOT.COM
The function y = f (x) = [x] is called the greatest integer function, where [x] equals to the
greatest integer less than or equal to x. For example :
for 1  x < 0 ; [x] =  1 ; for 0  x < 1 ; [x] = 0
for 1  x < 2 ; [x] = 1 ; for 2  x < 3 ; [x] = 2 and so on.

Properties of greatest integer function :

(a) x  1 < [x]  x

(b) If m is an integer, then [x ± m] = [x] ± m.

(c) [x] + [y]  [x + y]  [x] + [y] + 1

0 , if x is an int eger
(d) [x] + [ x] = 
  1 , if x is not an int eger

(ix) Fractional Par t Function:


It is defined as, y = {x} = x  [x], where [.] denotes greatest integer function.

60
e.g. the fractional part of the number 2.1 is 2.1  2 = 0.1 and {–3.7} = 0.3.
The period of this function is 1 and graph of this function is as shown.

(x) dentity function :


The function f : A  A defined by, f(x) = x,  x  A is called
the identity function on A and is denoted by A. It is easy to
observe
that identity function is a bijection.

(xi) Constant function :


A function f : A  B is said to be a constant function, if every
element of A has the same f image in B. Thus f : A  B;
f(x) = c,  x  A, c  B is a constant function.
Example # 8 : (i) Let {x} and [x] denote the fractional and integral part of a real number x respectively.
Solve 4{x} = x + [x]
(ii) Draw graph of f(x) = sgn (n x)
Solution : (i) As x = [x] + {x}
2 [ x]
 Given equation  4{x} = [x] + {x} + [x]  {x} =
3
As [x] is always an integer and {x}  [0, 1), possible values are
[x] {x} x = [x] + {x}
0 0 0

STUDYPIVOT.COM
1
2
3
5
3
5
 There are two solution of given equation x = 0 and x =
3

(ii)

Self practice problems :


(5) If f : R  R satisfying the conditions f(0) = 1, f(1) = 2 and f(x + 2) = 2f (x) + f(x + 1), then find
f (6).
(6) Draw the graph of following functions, where [.] denotes greatest integer function
(i) y= [2 x] + 1 (ii) y = x [x], 1  x  3 (iii) y = sgn (x 2 – x)
Answers : (5) 64

(6) (i) (ii)

61
(iii)

Equal or dentical Functions :


Two functions f and g are said to be identical (or equal) iff :
(i) The domain of f  the domain of g.
(ii) f(x) = g(x), for every x belonging to their common
domain.

1 x
e.g. f(x) = and g(x) = 2 are identical functions.
x x
Clearly the graphs of f(x) and g(x) are exactly same

x2
But f(x) = x and g(x) = are not identical functions.
x

Clearly the graphs of f(x) and g(x) are different at x = 0.

(i)
STUDYPIVOT.COM
Example # 9 : Examine whether following pair of functions are identical or not ?

f(x) =
x2  1
and g(x) = x + 1
x 1
(ii) f(x) = sin2x + cos2x and g(x) = sec 2x – tan2x

Solution : (i) No, as domain of f(x) is R – {1}


while domain of g(x) is R
(ii) No, as domain are not same. Domain of f(x) is R

  
while that of g(x) is R – 2n  1 ; n  I
 2 
Self practice problems
(7) Examine whether the following pair of functions are identical or not :

 x
 x0
(i) f(x) = sgn (x) and g(x) =  | x |
 0 x0


(ii) f(x) = sin–1x + cos –1x and g(x) =
2
Answers : (i) Yes (ii) No

Composite Function :

Let f: XY1 and g: Y2 Z be two functions and D is the set of values of x such that if x  X, then f(x)  Y2.
If D  , then the function h defined on D by h(x) = g{f(x)} is called composite function of g and f and is
denoted by gof. It is also called function of a function.
62
Note :  Domain of gof is D which is a subset of X (the domain of f ). Range of gof is a subset of the range of
g. If D = X, then f(X) Y2.

Pictorially gof(x) can be viewed as under

Note that gof(x) exists only for those x when range of f(x) is a subset of domain of g(x).
Properties of Composite Functions :
(a) In general gof  fog (i.e. not commutative)
(b) The composition of functions are associative i.e. if three functions f, g, h are such that
fo (goh) and (fog) oh are defined, then fo (goh) = (fog) oh.

Example # 10 : Describe fog and gof wherever is possible for the following functions
(i) f(x) = x  3 , g(x) = 1 + x 2 (ii) f(x) = x , g(x) = x  1.
2

Solution : (i) Domain of f is [3, ), range of f is [0, ).


Domain of g is R, range of g is [1, ).
For gof(x)
Since range of f is a subset of domain of g,
 domain of gof is [3, ) {equal to the domain of f }
gof (x) = g{f(x)} = g ( x  3 ) = 1 + (x+3) = x + 4. Range of gof is [1, ).
For fog(x)
since range of g is a subset of domain of f,
 domain of fog is R {equal to the domain of g}
fog (x) = f{g(x)}= f(1+ x 2 ) = x 2  4 Range of fog is [2, ).
(ii)
STUDYPIVOT.COM
f(x) = x , g(x) = x  1.
2

Domain of f is [0, ), range of f is [0, ).


Domain of g is R, range of g is [1, ).
For gof(x)
Since range of f is a subset of the domain of g,
 domain of gof is [0, ) and g{f(x)}= g(x) = x  1. Range of gof is [1, )
For fog(x)
Since range of g is not a subset of the domain of f
i.e. [1, )  [0, )
 fog is not defined on whole of the domain of g.
Domain of fog is {xR, the domain of g : g(x) [0, ), the domain of f}.
Thus the domain of fog is D = {xR: 0  g(x) < }
i.e. D = { xR: 0  x 2  1}= { xR: x  1 or x  1 }= (, 1]  [1, )
fog (x) = f{g(x)} = f(x21) = x 2  1 Its range is [0, ).

  
Example # 11 : Let f(x) = ex ; R+  R and g(x) = sin–1 x; [–1, 1]   ,  . Find domain and range of fog(x)
 2 2
  
Solution : Domain of f(x) : (0, ) Range of g(x) :  , 
 2 2
 
values in range of g(x) which are accepted by f(x) are  0, 
 2
 
 0 < g(x)   0 < sin–1x   0<x1
2 2
Hence domain of fog(x) is x  (0, 1]
Therefore Domain : (0, 1]
Range : (1, e /2]
63
Example # 12 : Composition of piecewise defined functions :
If f(x) = | |x – 3| – 2 | 0x4
g(x) = 4 – |2 – x| –1x3
then find fog(x) and draw rough sketch of fog(x).
Solution : f(x) = | | x – 3| – 2| 0  x  4

1  x 0  x  1
| x  1 | 0  x  3 
=  = x 1 1 x  3
| x  5 | 3  x  4 5  x 3  x  4

g(x) = 4 – |2 – x| 1  x  3

4  ( 2  x )  1  x  2 2  x  1  x  2
=  = 
4  ( x  2) 2  x  3 6  x 2  x  3

1  ( 2  x ) 0  2  x  1 and  1  x  2

1  g( x ) 0  g( x )  1  2  x 1 1  2  x  3 and  1  x  2
 5  (2  x ) 3  2  x  4 and  1  x  2
g( x )  1 1  g( x )  3
 fog (x) =  = 
5  g( x ) 3  g( x )  4  1 6  x 0  6  x  1 and 2x3

 6  x 1 1  6  x  3 and 2x3

 5  6  x 3  6  x  4 and 2x3
 1  x  2  x  1 and  1  x  2   1  x  2  x  1 and  1  x  2
 
 1 x 1 x  1 and  1  x  2  1 x 1 x  1 and  1  x  2
 3  x 1 x  2 and 1 x  2 
3x 1 x  2 and  1  x  2
=  = 
 x  5  6   x  5 and 2x3  x  5 5 x6 and 2x3


STUDYPIVOT.COM
 5  x  5   x  3
 x  1  3   x  2
and
and
2x3
2x3


5x
 x  1
3x5
2x3
and
and
2x3
2x3

1  x  1  x  1

= 3  x 1  x  2
x 1 2  x  3

Alternate method for finding fog

2  x  1  x  2
g(x) = 
6  x 2  x  3
graph of g(x) is

1  g( x ) 0  g( x )  1

g( x )  1 1  g( x )  3
 fog(x) = 
5  g( x ) 3  g( x )  4

64
1  g( x ) for no value  2  x 1 1 x  1 x 1 1 x  1
  
g( x )  1  1  x  1
=  = 5  (2  x ) 1  x  2 = 3  x 1  x  2
5  g( x ) 1 x  3 5  (6  x ) 2  x  3 x 1 2  x  3
  
Self practice problems
(8) Define fog(x) and gof(x). Also find their domain and range.
(i) f(x) = [x], g(x) = sin x (ii) f(x) = tan x, x  (–/2, /2); g(x) = 1 x 2

(9) Let f(x) = ex : R+  R and g(x) = x 2 – x : R  R. Find domain and range of fog (x) and gof (x)

Answers :
(8) (i) gof = sin [x] domain : R range { sin a : a  }
fog = [ sin x] domain : R range : {–1, 0, 1}

  
(ii) gof  1  tan 2 x , domain :  ,  range : [0, 1]
 4 4

fog  tan 1 x 2 domain : [–1, 1] range [0, tan 1]

(9) fog (x) gof (x)


Domain : (–, 0)  (1, ) Domain : (0, )
Range : (1, ) Range : (0, )

Classification of Functions :
Functions can be classified as "One  One Function (Injective Mapping)" and "Many  One Function":

One - One Function : STUDYPIVOT.COM


A function f : A  B is said to be a one-one function or injective mapping if different elements of A
have different f images in B.
Thus for x1, x2  A and f(x1), f(x 2)  B, f(x1) = f(x 2)  x 1 = x 2 or x 1  x 2  f(x 1)  f(x 2).
Diagrammatically an injective mapping can be shown as

OR

Many - One function :


A function f : A  B is said to be a many one function if there exist at least two or more elements of A
having the same f image in B.
Thus f : A  B is many one iff there exist atleast two elements x 1, x 2  A, such that f(x 1) = f(x2) but
x 1  x 2.

Diagrammatically a many one mapping can be shown as

OR

Note :  If a function is oneone, it cannot be manyone and vice versa.

65
Methods of determining whether a given function is ONE-ONE or MANY-ONE :
(a) If x 1, x2  A and f(x 1), f(x 2)  B, equate f(x 1) and f(x2) and if it implies that x 1 = x 2, then and
only then function is ONE-ONE otherwise MANY-ONE.
(b) If there exists a straight line parallel to x-axis, which cuts the graph of the function atleast at
two points, then the function is MANY-ONE, otherwise ONE-ONE.
(c) If either f(x)  0,  x  domain or f(x)  0  x  domain, where equality can hold at discrete
point(s) only i.e. strictly monotonic, then function is ONE-ONE, otherwise MANY-ONE.
Note : If f and g both are one-one, then gof and fog would also be one-one (if they exist). Functions can also
be classified as "Onto function (Surjective mapping)" and "Into function":

Onto function :
If the function f : A  B is such that each element in B (codomain) must have atleast one
preimage in A, then we say that f is a function of A 'onto' B. Thus f : A  B is surjective iff  b  B,
there exists some a  A such that f (a) = b.
Diagrammatically surjective mapping can be shown as

OR

Into function :
If f : A  B is such that there exists atleast one element in codomain which is not the image of any
element in domain, then f(x) is into.

STUDYPIVOT.COM
Diagrammatically into function can be shown as

OR

Note : (i) If range  codomain, then f(x) is onto, otherwise into


(ii) If a function is onto, it cannot be into and vice versa.

A function can be one of these four types:

(a) oneone onto (injective and surjective)

(b) oneone into (injective but not surjective)

(c) manyone onto (surjective but not injective)

(d) manyone into (neither surjective nor injective)

Note : (i) If f is both injective and surjective, then it is called a bijective mapping. The bijective functions
are also named as invertible, non singular or biuniform functions.
(ii) If a set A contains 'n' distinct elements, then the number of different functions defined from
A  A is nn and out of which n! are one one.
(iii) If f and g both are onto, then gof or fog may or may not be onto.
66
(iv) The composite of two bijections is a bijection iff f and g are two bijections such that gof is
defined, then gof is also a bijection only when co-domain of f is equal to the domain of g.

Example # 13 : (i) Find whether f(x) = x + cos x is one-one.


3 2
(ii) Identify whether the function f(x) = –x + 3x – 2x + 4 for f : R  R is ONTO or INTO
(iii) f(x) = x2 – 2x + 3; [0, 3]  A. Find whether f(x) is injective or not. Also find the set A,
if f(x) is surjective.
Solution : (i) The domain of f(x) is R. f (x) = 1  sin x.
 f (x)  0  x  complete domain and equality holds at discrete points only
 f(x) is strictly increasing on R. Hence f(x) is one-one.
(ii) As range  codomain, therefore given function is ONTO

(iii) f(x) = 2(x – 1); 0  x  3

 ve ; 0  x  1
 f(x) = 
 ve ; 1  x  3

 f(x) is non monotonic. Hence it is not injective.


For f(x) to be surjective, A should be equal to its range. By graph range is [2, 6]
 A  [2, 6]

Self practice problems :

(10) For each of the following functions find whether it is one-one or many-one and also into or
onto
1
(i) f(x) = 2 tan x; (/2, 3/2)  R (ii) f(x) = ; (–, 0)  R

(iii) STUDYPIVOT.COM
f(x) = x2 + n x
1 x2

Answers : (i) one-one onto (ii) one-one into (iii) one-one onto

Odd and Even Functions :


(i) If f (x) = f (x) for all x in the domain of ‘f’, then f is said to be an even function.
e.g. f (x) = cos x; g (x) = x² + 3.
(ii) If f (x) = f (x) for all x in the domain of ‘f’, then f is said to be an odd function.
e.g. f (x) = sin x; g (x) = x 3 + x.
Note : (i) A function may neither be odd nor even. (e.g. f(x) = ex , cos –1x)
(ii) If an odd function is defined at x = 0, then f(0) = 0

Properties of Even/Odd Function


(a) The graph of every even function is symmetric about the yaxis and that of every odd function
is symmetric about the origin.
For example graph of y = x 2 is symmetric about y-axis, while graph of y = x 3 is symmetric
about origin

(b) All functions (whose domain is symmetrical about origin) can be expressed as the sum of an
even and an odd function, as follows

f(x) =

67
(c) The only function which is defined on the entire number line and is even and odd at the same
time is f(x) = 0.

(d) If f and g both are even or both are odd, then the function f.g will be even but if any one of
them is odd and the other even then f.g will be odd.

(e) If f(x) is even then f(x) is odd while derivative of odd function is even. Note that same cannot
be said for integral of functions.

Example # 14 : Show that log  x  x 2  1  is an odd function.


 
Solution :  2 
Let f(x) = log  x  x  1  .
 

Then f(–x) = log   x  (  x )2  1 


 
 

  2  2 
  x  1  x  x  1  x  1
= log   = log = – log  x  x 2  1  = –f(x)
 x2  1  x  2
x 1 x  
 

or f(x) + f(–x) = 0
Hence f(x) is an odd function.
Example # 15 : Show that ax +a–x is an even function.
Solution : Let f(x) = ax + a–x
Then f(–x) = a–x + a–(–x) = a–x +ax = f(x).
Hence f(x) is an even function

Solution : STUDYPIVOT.COM
Example # 16 : Show that cos –1 x is neither odd nor even.
Let f(x) = cos –1x. Then f(–x) = cos –1 (–x) =  – cos–1 x which is neither equal to f(x) nor equal
to – f(x).
Hence cos –1 x is neither odd nor even

Self practice problems

(11) Determine whether the following functions are even or odd?

e x  e x  2 
(i) (ii) log  x  1  x 
x
e e x  

 2 
(iii) x log  x  x  1  (iv) sin–1 2x 1 x 2
 
Answers (i) Odd (ii) Odd
(iii) Even (iv) Odd

Even extension / Odd extension :


Let f(x) be defined in [a, b] where ab  0. Even extension of this function implies to define the function
in [–b, –a] to make it even. In order to get even extension replace x by –x in the given definition.
Similarly, odd extension implies to define the function in [–b, –a] to make it odd. In order to get odd
extension, multiply the definition of even extension by –1

Example # 17 : What is even and odd extensions of f(x) = x 3 – 6x2 + 5x – 11, x  0


Solution : Even extension of f(x) :
f(–x) = –x 3 – 6x2 – 5x – 11 ;x<0
Odd extension of f(x) :
– f(–x) = x 3 + 6x2 + 5x + 11 ;x<0

68
Periodic Functions :
A function f(x) is called periodic with a period T if there exists a real number T > 0 such that for each
x in the domain of f the numbers x – T and x + T are also in the domain of f and f(x) = f(x + T) for all
x in the domain of f(x). Graph of a periodic function with period T is repeated after every interval of
'T'.
e.g. The function sin x and cos x both are periodic over 2 and tan x is periodic over 
The least positive period is called the principal or fundamental period of f(x) or simply the
period of the function.

Note : Inverse of a periodic function does not exist.

Properties of Periodic Functions :


1
(a) If f(x) has a period T, then and f( x ) also have a period T.
f( x )

T
(b) If f(x) has a period T, then f (ax + b) has a period | a | .

(c) Every constant function defined for all real x, is always periodic, with no fundamental period.
(d) If f (x) has a period T 1 and g (x) also has a period T 2 then period of f(x) ± g(x) or f(x) . g(x) or
f (x)
is L.C.M. of T 1 and T 2 provided their L.C.M. exists. However that L.C.M. (if exists) need
g( x )

f (x)
not to be fundamental period. If L.C.M. does not exists then f(x) ± g(x) or f(x) . g(x) or is
g( x )
nonperiodic.

STUDYPIVOT.COM
 a p   L.C.M.(a, p,  )
L.C.M. of  , ,  = H.C.F. (b, q, m)
 b q m
e.g. |sinx| has the period , | cosx | also has the period 

 |sinx| + |cosx| also has a period . But the fundamental period of |sinx| + |cosx| is .
2
(e) If g is a function such that gof is defined on the domain of f and f is periodic with T, then gof
is also periodic with T as one of its periods.

Example # 18 : Find period of the following functions


x x
(i) f(x) = sin + cos (ii) f(x) = {x} + sin x, where {.}denotes fractional
2 3
part function
3x x 2x
(iii) f(x) = cos x . cos 3x (iv) f(x) = sin – cos – tan
2 3 3

x x x x
Solution : (i) Period of sin is 4 while period of cos is 6 . Hence period of sin + cos
2 3 2 3
is 12 
{L.C.M. of 4 and 6 is 12}
(ii) Period of sin x = 2
Period of {x} = 1
but L.C.M. of 2 and 1 is not possible as their ratio is irrational number
 it is aperiodic
(iii) f(x) = cos x . cos 3x
 2 
period of f(x) is L.C.M. of  2,  = 2
 3 
69
2
but 2 may or may not be fundamental periodic, but fundamental period = , where
n
n  N. Hence cross-checking for n = 1, 2, 3, ....we find  to be fundamental period
f( + x) = (– cos x) (– cos 3x) = f(x)
2 2  4 3
(iv) Period of f(x) is L.C.M. of , , = L.C.M. of , 6 , = 12
3 / 2 1/ 3 2 / 3 3 2

Self practice problems :


(12) Find the period of following function.
x
(i) f(x) = sin x + | sin x | (ii) f(x) = 3 cos x – sin 3

2x 3x
(iii) sin – cos (iv) f(x) = sin2x + cos4x
5 7
Answers : (i) 2 (ii) 6 (iii) 70  (iv) 

Inverse of a Function :
Let y = f(x) : A  B be a one-one and onto function. i.e. bijection, then there will always exist bijective
function x = g(y) : B  A such that if (p, q) is an element of f, (q, p) will be an element of g and the
functions f(x) and g(x) are said to be inverse of each other. g(x) is also denoted by f 1(x) and f(x) is
–1
denoted by g (x)
Note : (i) The inverse of a bijection is unique.
(ii) Inverse of an even function is not defined.

Properties of Inverse Function :


(a) STUDYPIVOT.COM
The graphs of f and g are the mirror images of each other in the line y = x. For example f(x) =
ax and g(x) = loga x are inverse of each other, and their graphs are mirror images of each
other on the line y = x as shown below.

(b) Normally points of intersection of f and f –1 lie on the straight line y = x. However it must be
–1
noted that f(x) and f (x) may intersect otherwise also. e.g f(x) = 1/x

(c) In general fog(x) and gof(x) are not equal. But if f and g are inverse of each other, then gof =
fog. fog(x) and gof(x) can be equal even if f and g are not inverse of each other. e.g. f(x) = x
–1
+ 1, g(x) = x + 2. However if fog(x) = gof(x) = x, then g(x) = f (x)

(d) If f and g are two bijections f : A  B, g : B  C, then the inverse of gof exists and
(gof) 1 = f 1 o g1.
1
(e) If f(x) and g(x) are inverse function of each other, then f(g(x)) = g( x )

2x  3
Example # 19 : (i) Determine whether f(x) = for f : R  R, is invertible or not? If so find it.
4
–1  2x 1  x 2 
(ii) Is the function f(x) = sin   invertible?
 
70
(iii) Let f(x) = x 2 + 2x; x  –1. Draw graph of f –1(x) also find the number of solutions of the
equation, f(x) = f –1(x)
(iv) If y = f(x) = x 2 – 3x + 1, x  2. Find the value of g(1) where g is inverse of f
Solution : (i) Given function is one-one and onto, therefore it is invertible.
2x  3
y=
4
4y  3 –1 4x  3
 x=  f (x) =
2 2
(ii) Domain of f is [–1, 1]
f(0) = 0 = f(1)
 f is not one – one  f is not invertible

(iii)

f(x) = f –1(x) is equivalent to f(x) = x


 x 2 + 2x = x  x(x + 1) = 0  x = 0, –1

(iv)
STUDYPIVOT.COM
Hence two solution for f(x) = f –1(x)

y= 1  x 2 – 3x + 1 = 1
 x (x – 3) = 0  x = 0, 3
But x2  x=3
Now g(f(x)) = x
Differentiating both sides w.r.t. x

1
 g(f(x)). f(x) = 1  g(f(x)) =
f ( x )

1 1 1
 g(f(3)) =  g (1) = = = (As f(x) = 2x – 3)
f (3) 63 3

Alternate Method

y = x 2 – 3x + 1
x 2 – 3x +1 – y = 0

3  9  4(1  y )
x=
2
3  5  4y
=
2
x2

3  5  4y
x=
2

3  5  4x
g(x) =
2
71
1
g(x) = 0 + 4
4 5  4x

1 1 1
g(1) = = =
54 9 3

Self practice problems :

–1
(13) Determine f (x), if given function is invertible
2
(i) f : (–, –1)  (–, –2) defined by f(x) = –(x + 1) – 2

  7   
(ii) f:  ,   [–1, 1] defined by f(x) = sin  x  
6 6   3

2 –1
Answers : (i) –1– x2 (ii) + sin x
3

General tips :
If x, y are independent variables, then:
(i) f (xy) = f (x) + f (y)  f (x) = k ln x or f (x) = 0.
(ii) f (xy) = f (x). f (y)  f (x) = x n, n  R
(iii) f (x + y) = f (x). f (y)  f (x) = akx.
(iv) f (x + y) = f (x) + f (y)  f(x) = kx, where k is a constant.

STUDYPIVOT.COM
 1  1
Example # 20 : If f(x) is a polynomial function satisfying f(x) . f   = f(x) + f    x  R – {0} and f(2) =9,
x x
then find f (3)
Solution : f(x) = 1 ± x n
As f(2) = 9  f(x) = 1 + x3
3
Hence f(3) = 1 + 3 = 28

Self practice problems

 1  1
(14) If f(x) is a polynomial function satisfying f(x) . f   = f(x) + f    x  R – {0} and
x
  x
f(3) = – 8, then find f(4)

f (x)
(15) If f(x + y) = f(x) . f(y) for all real x, y and f(0)  0, then prove that the function, g(x) =
1  f 2 (x)
is an even function.

Answer : (14) – 15

72
EXERCISE - 1: IMPORTANT BOARD LEVEL QUESTIONS
Type (I) : Very Short Answer Type Questions : [01 Mark Each]

1. Let f, g be the functions f = {(1, 5), (2. 6), (3, 4)}, g = {(4, 7), (5, 8), (6, 9). What is the range of f and g.
| x – 1|
2. What is the range of the function f(x) = ?
( x – 1)
Type (II) : Short Answer Type Questions : [02 Marks Each]

3. State whether the function f : N  N given by f(x) = 5x is injective surjective or both.

4. If f : R  R is defined by f(x) = 3x + 2, find f(f(x)).

Type (III) : Long Answer Type Questions: [04 Mark Each]


x 1
5. If f(x) = (x  1, – 1), show that fof–1 is an identity funciton.
x 1
6. Show that the function f : Z  Z given by f(x) = x2 is not injective.

7. Find the inverse of the function f(x) = 4x – 7, x  R.


8. Let f : x  y and g : y  z be two invertible functions.
Then prove that gof is also invertible with (gof) –1 = f–1 og–1

Type (IV) : Very Long Answer Type Questions: [06 Mark Each]
x3
9. If the function f : R  R is given by f(x) = and g : R  R is given by g(x) = 2x – 3, find (i) fog and (ii) gof.
2
Is f = g ?
STUDYPIVOT.COM
–1

4x  3 2 2
10. If f(x) = , x  , show that fof(x) = x for all x  . What is the inverse of f ?
6x – 4 3 3

1
13. The domain of the function f(x) = is : [AIEEE 2011]
| x | x
(1) (– , ) (2) (0, ) (3) (–, 0) (4) (–, ) – {0}

14. Let f be a function defined by f(x) = (x–1)2 + 1, (x  1). [AIEEE 2011]


Statement - 1 : The set {x : f(x) = f–1(x)} = {1, 2}.
Statement - 2 : f is a bijection and f–1(x) = 1 + x  1 , x  1.

(1) Statement-1 is true, Statement-2 is true; Statement-2 is a correct explanation for Statement-1.
(2) Statement-1 is true, Statement-2 is true; Statement-2 is NOT a correct explanation for Statement-1
(3) Statement-1 is true, Statement-2 is false
(4) Statement-1 is false, Statement-2 is true .

73
EXERCISE - 2: CBSE PREVIOUS YEARS QUESTIONS

1. Consider f : N  N, g : N  N and h : N  R defined as f(x) = 2x, g(y) = 3y + 4 and h(z) = sin z  x, y and
z in N. Show that ho (gof) = (hog)o f. [CBSE 2004]

2. Let y = {n2 : n N} Consider f : N  y as f(n) = n2. Show that f is invertible. Find the inverse of f.
[CBSE 2005]
3. Let f : N  Y be a function defined as f(x) = 4x + 3, where Y = {y N : y = 4x + 3 for some x  N}.
Show that f is invertible. Find the inverse. [CBSE 2005]

4. Let A = {1, 2, 3} and B = {4, 5, 6} f : A  B is a function defined as f(1) = 4, f(2) = 5, f(3) = 6. Write down
f–1 as a set of ordered pairs. [CBSE 2005]

5. Let f : R  R be defined as f(x) = x4. Show that f is not one-one. [CBSE 2006]

6. If f be a greatest integer function and g be an absolute value function. Find the value of
–3 4
(fog)   + (gof)   . [CBSE 2007]
 2  3

7. If f : R  R is defined by f(x) = x2 – 3x + 2, find f(f(x)). [CBSE 2007]

8. If f(x) = x + 7 and g(x) = x – 7, x  R, find (fog) (7). [CBSE 2008]

3x  5
9. If f : R  R defined by f(x) = is an invertible function, find f–1. [CBSE 2009, 2008]
2

STUDYPIVOT.COM
10. If the function f : R  R is given by f(x) = x2 + 3x + 1 and g : R  R is given by g(x) = 2x – 3, find
(i) fog and (ii) gof. [CBSE 2009, 2008]

11. If f : R  R be defined by f(x) = (3 – x3)1/3, then find fof(x). [CBSE 2010, 2006]

12. Let A = {1, 2, 3}, B = {4, 5, 6, 7} and let f = {(1, 4), (2, 5), (3, 6)} be a function from A to B. State whether f is
one-one or not. [CBSE 2011, 2005]
13. Show that the function f : W  W defined by [CBSE 2012, 2011, 2009]
n  1 , if n is even
f(n) =  is a bijective function.
n – 1 , if n is odd
x – 2
14. Let A = R – {3} and B = R – {1}. Consider the function f : A  B defined by f(x) =   . Show that f is one-
x–3
one and onto and hence find f–1. [CBSE 2012, 2006]
2 4x  3
15. Show that the function f in A = R –   defined as f(x) = is one-one and onto. Hence find f–1 .
3  6x – 4
[CBSE 2013]

74
SOLUTIONS
EXERCISE - 1  f(x) is onto.  f (x) = 4x – 7 is invertible
1. Range of f = {4, 5, 6}  f–1 exists
Range of g = {7, 8, 9} For xR , Let f–1 (x) = y
 f(y) = x  4y – 7 = x
2.  Range = {–1, 1} x7 x7
 y= Hence f–1(x) =
4 4
3. f(x) = 5x is injective (one-one)
Explanation : The function f is one-one, for 8. To show that gof is invertible with (gof)–1 = f–1 og–1
f(x1) = f(x2) It is enough to show that
 5x1 = 5x2  x 1 = x2 (i) (f–1 og–1) o (gof) = Ix and
Further f is not onto, since for 1  N there does not (ii) (gof) o (f–1 og–1) = Iy.
exist any x  N such that f(x) = 5x = 1
Now (f–1 og–1) o (gof) = (f–1 og–1)og) of
 f is one-one (injective) but not one one onto (bijec-
[By associative Law]
tive).
= (f–1o(g–1 og)) of [By associative Law]
= (f–1 o Iy) of [By definition of g–1]
4. We have f(x) = 3x + 2
 f(f(x)) = 3(3x + 2) + 2 = 9x + 6 + 2 = 9x + 8 = f–1 of [Identity Law]
= Ix [by definition of f–1]
x 1 Again (f og ) o (gof) = (f og–1)og) of
–1 –1 –1
5. Let y =
x 1 [By associative Law]
 xy + y = x – 1  xy – x = –y – 1 = (f–1 o) g–1 og) of [By associative Law]
1 y = (f–1 o Iy) of [By definition of g–1]
 x = 1 y [ y = f(x)  x = f–1 (y)] –1
= (f o f) [Identity Law]
= Ix [by definition of f–1]
1 x

STUDYPIVOT.COM
–1 –1 –1
 f–1 (x) = Hence (gof) = f o g
1 x
 1 x 
Now fof–1 (x) = f(f–1(x)) = f   2x  3  3 2 x
 1 x  9. (i) (fog)x = f(g(x)) = f(2x – 3) = = =x
2 2
1 x
1
1 x  x  1 x 3
= 1 x  f ( x )  x  1 (ii) (gof) x = g(f(x)) = g  
1    2 
1 x
x3
1  x  (1  x ) 2x = 2  –3=x+3–3=x
= = = x =  (x)  x  2 
1  x  (1  x ) 2
 fof–1 =  (identity function) Since fog = gof = R
Hence f–1 = g
6. Let x1, x2  Z be such that f(x1) = f(x2)
 x12 = x 22  x12 – x 22 = 0 4x  3 2
10. Given f(x) = ,x
 (x1 + x2) (x1 – x2) = 0  x1 = – x2 or x1 = x2 6x – 4 3
(f o f) (x) = f { f (x) }
 f(x1) = x12 and f(– x1) = x12
 4x  3 
 f(x) = f(– x) for all x1  Z
 f is not one-one
=   6x – 4 
Hence f is not injective.
 4x  3 
4 3
7. We have f(x) = 4x – 7, xR  6x – 4 
=
Let x1, x2  R  4x  3 
6 –4
f(x1) = f(x2)  4x1 – 7 = 4x2 – 7  6x – 4 
 4x1 = 4x2  x 1 = x2
 f(x) is one-one. 16 x  12  18 x – 12 34 x
=  =x
Let y  R 24 x  18 – 24 x  16 34
f(x) = y (f o f) (x) = x f–1 = f
y7
4x – 7 = y  x = R
4
75
EXERCISE - 2 –3  –3  3 
 (fog)   = 
2
 =   =1
 2    2
CBSE PREVIOUS YEARS (gof) x = g(f(x)) = g([x]) = | [x] |
1. We have f(x) = 2x, g(y) = 3y + 4 and h(z) = sin z
L.H.S. = ho(gof) (x) = h(g(f(x)) = h(g(2x)) 4 4
 (gof)   =  3  = |1| = 1
= h(3(2x) + 4) = h(6x + 4) = sin (6x + 4) 3
   
R.H.S. = (hog) of (x) = (hog) (f(x)) = (hog)(2x)
= h(g(2x)) = h(3(2x) + 4) = h(6x + 4) –3 4
(fog)   + (gof)   = 1 + 1 = 2
= sin (6x + 4)  2  3
 L.H.S. = R.H.S.
7. f(f(x)) = f(x2 – 3x + 2)
2. by the definition y, we have = (x2 – 3x + 2)2 – 3(x2 – 3x + 2) + 2
= x4 + 9x2 + 4 – 6x3 – 12x + 4x2 – 3x2 + 9x – 6 +
y = f(n) = n2  n = y . For some n  N
2
Now define g : y  N by g(y) = = x4 – 6x3 + 10x2 – 3x
y
8. Here f(x) = x + 7, g(x) = x – 7
Now gof (n) = g(f(n)) = g(n2) = n 2 = n = In
(fog)x = f(g(x))
2
and fog(y) = f(g(n)) = f(g(y)) = f ( y ) = ( y ) = y = Iy = f(x – 7)
=x–7+7=x  (fog) (7) = 7
 f–1 = g and f–1(x) = x 3x  5
9. Let f(x) = y  y=
2
3. Consider y  Y be arbitrary.
By definition of Y, Since f(x) is invertible  x = f–1(y)
y = 4x + 3 for some x in the domain N.  3x + 5 = 2y

y–3 2y – 5
 x= 3x = 2y – 5  x=
4 3

STUDYPIVOT.COM
Now define g : Y  N by g(y) =
y–3
4
 f–1(y) =
2y – 5
3
Now, gof(x) = g(f(x)) = g(4x + 3) 2x – 5
Hence f–1(x) =
4x  3 – 3 4x 3
= = = x = IN 11. Here f(x) = (3 – x3)1/3
4 4
 (fof) x = f(f(x) = f((3 – x3)1/3)
y–3 4( y – 3) = [3 – ((3 – x3)1/3)3]1/3
and fog (x) = f(g(y)) = f   = +3
 4  4 = (3 – 3 + x3)1/3 = (x3)1/3 = x
= y – 3 + 3 = y = Iy
 f is invertible and g is the inverse of f. 12. f = {(1, 4), (2, 5), (3, 6)}
Hence f–1 = g. f(1) = 4, f(2) = 5, f(3) = 6.
Different elements of the domain have different
4. We have f(1) = 4. f(2) = 5 and f(3) = 6 f-images in the range.
 f is one-one
 The image of distinct elements in A are distinct
 f is one-one
Also every element in B has at least one pre-image n  1 , if n is even
 f is onto, f is invertible i.e. f–1 exists 13. f(n) = 
n – 1 , if n is odd
Now define f–1 : B  A as (a) f is onto
f–1(4) = 1  f(1) = 4, f–1(5) = 2 and f–1(6) = 3 Let n  W (= codomain)
 f–1 = {(4, 1), (5, 2), (6, 3)} If n = 2m, then f(2m +1) = (2m +1) – 1 = 2m = n
If n = 2m + 1, then f(2m) = 2m + 1 = n
5. Here f(x) = x4 Also f(1) = 0
 f(–x) = (–x)4 = x4 = f(x)  f : W  W is onto
 f(–x) = f(x) for all x  R (b) f is one-one
Therefore, f is not one-one let f(n1) = f(n2), n1, n2  W
If n1 and n2 either both are even or both are odd, then
6. Here f(x) =7 [x] and g(x) = |x|  f(n1) = f(n2)  n1 + 1 = n2 + 1 (if both are even)
(fog)x = f(g(x)) = f(|x|) = [|x|]  n1 = n2
Also f(n1) = f(n2)  n1 – 1 = n2 – 1 (if both are odd)
76
 n1 = n2 So, f is onto.
In the case when one of n1 and n2 is even and the 4y  3 4y  3
other is odd or vice-versa, we can’t have f(n1) = f(n2) Here x = 6 y – 4  f–1(y) = 6 y – 4
 f is one-one
Hence f : W  W is bijection. 4x  3
Hence, f–1(x) =
x–2 6x – 4
14. Given, A = R – (3), B = R – [1] and f(x) =
x–3
For one-one : Let fo x1, x2  A
f(x1) = f(x2)
x1 – 2 x2 – 2
 x1 – 3 = x2 – 3
 x1x2 – 2x2 – 3x1 + 6 = x1x2 – 3x2 – 2x1+ 6
 x 2 = x1
As f(x1) = f(x2)  x1 = x2
Hence, function is one-one
For onto : Let for y  B, there exists x  A such that
y = f(x)
x–2
y=
x–3
 xy – 3y = x – 2  xy – x = 3y – 2
 x(y – 1) = 3y – 2
3y – 2
 x= A ...(i)
y –1
Hence onto
3y – 2
For inverse, x =
y –1

STUDYPIVOT.COM
3y – 2
 f–1(y) = [From (i)]
y –1
3x – 2
 f–1(x) =
x –1
4x  3
15. Here f(x) =
6x – 4
2
One - One: Let x, y  R   such that f(x) = f(y)
3 
4x  3 4y  3
 
6x – 4 6 y – 4
 (4x + 3) (6y – 4) = (4y + 3) (6x – 4)
 24xy + 18y – 16x – 12 = 24xy + 18x – 16y – 12
 18y –16x = 18x – 16y
 34y = 34x  x=y
2
Thus f(x) = f(y)  x = y for all x, y R –  
3 
So, f is one-one map
2
Onto : Let y be an arbitrary element of R –   .
3 
Then
4y  3
f(x) = y  = y 4x + 3 = y = y(6x – 4)
6x – 4
4y  3
 4x+ 3 = (6y – 4)x  x = 6 y – 4

2
Thus, for every y R –  
3 
2 77
there exists x R –  
3 
such that f(x) = y.

Das könnte Ihnen auch gefallen